You are on page 1of 44

TYPE

BREAK UP OF QUESTIONS

S.
Topic Sub Topic Questions
No.
Nature and Scope of Science & Technology 0
Relevance of Science & Technology to the day to day life 5
Science and National Policy on Science, Technology and Innovation 2
1
Technology: Institutes and Organization in India promoting integration of Science 5
Technology and Innovation, their activities and contribution 3
Contribution of Prominent Indian Scientists. 0
Nature and Scope of ICT 0
Information ICT in day to day life 4
and ICT and Industry 5
2 Communication ICT and Governance - Various government schemes promoting use of ICT, E-
Technology 7
Governance programmes and services
(ICT) Cyber Security Concerns - National Cyber Crime Policy, Netiquettes 4
Miscellaneous 5
Evolution of Indian Space Programme 1
Indian Space Research Organization (ISRO) — it's activities and achievements 8
Technology in Various Satellite Programmes — Satellites for Telecommunication, Indian
3
Space Regional Navigation Satellite System (IRNSS), Indian Remote Sensing (IRS) 4
Satellites; Satellites for defence, Eduset or Satellites for academic purposes
Miscellaneous 7
Defence Research and Development Organization (DRDO)- vision, mission and
11
4 Defence activities.
Miscellaneous 8
India's existing energy needs and deficit 1
India's Energy Resources and Dependence 1
5 Energy Energy policy of India Government Policies and Programmes 1
Solar, Wind and Nuclear energy 2
Issues and concerns related to environment 4
Biodiversity- its importance and concerns 6
Climate Change, International Initiatives (Policies, Protocols) and India's
commitment 4

6 Enironment Forest and Wildlife - Legal framework for Forest and Wildlife Conservation in
India 6

Environmental Hazards, pollution, carbon emission, Global warming 1


National Action plans on Climate Change and Disaster management 1
Miscellaneous 3
Biotechnology and Nanotechnology; Nature, Scope and application, Ethical,
Social, and Legal issues, Government Policies 5

Genetic Engineering; Issues related to it and its impact on human life. Health
7 Biotechology 6
& Environment

Miscellaneous

Grand Total 120


APPSC – GROUP-I, Prelims – TEST 20 (03-03-2024)
SCIENCE & TECHNOLOGY
1. Consider the following statements about powers the entire operation, is the world’s
Agnibaan SubOrbital Technological sole single piece 3D-printed engine. Hence
Demonstrator (SOrTeD): statement III is correct.
1) Agnibaan SOrTeD is a double-stage launch 2. Consider the following statements with
vehicle powered by Agnilet engine. respect to G-20 Digital Innovation Alliance
2) Agnilet engine is an entirely 3D-printed, Summit:
single-piece semi-cryogenic engine. I. It was organized by the Ministry of
3) It is capable of carrying payloads up to 100 Electronics and Information Technology
kg to an altitude of 700 km in five different (MeitY) in Delhi.
configurations. II. Its primary objective is to support startups
How many of the following statements are that develop innovative digital solutions
correct? that bridge the digital divide and propel
1) Only one 2) Only two global economic growth.
3) All three 4) None Which of the above given statement is/are
Answer: 2 correct?
Explanation: 1) I only 2) II only
 Agnibaan SOrTeD is a single-stage launch 3) Both I and II 4) Neither I nor II
vehicle powered by AgniKul’s patented Answer: 2
Agnilet engine. Hence statement I is Explanation:
incorrect.  The G20 Digital Innovation Alliance (G20-
 Agnibaan SOrTeD is an entirely 3D-printed, DIA) is a program that supports innovators
single-piece, 6 kilonewton (kN) semi who have digital solutions for various
cryogenic engine. Unlike traditional economies. The G20-DIA's primary
sounding rockets that launch from guide objective is to support startups that
rails, Agnibaan SOrTeD will lift off vertically develop innovative digital solutions that
and follow a predetermined trajectory to bridge the digital divide and propel global
perform a precisely orchestrated set of economic growth. Hence statement II is
manoeuvres during flight. Hence correct.
statement II is correct.  The Ministry of Electronics and Information
 It is a customisable launch vehicle that Technology (MeitY), Government of India,
could be launched in one or two stages. The and the MeitY Start-up Hub organised the
rocket stands 18 metres tall and has a mass G20 Digital Innovation Alliance Summit
of 14,000 kg. It is capable of carrying in Bengaluru at the Bangalore Palace, from
payloads up to 100 kg to an altitude of 700 August 17th to 19th, 2023. Hence
km in five different configurations. The statement I is incorrect.
rocket's first stage could have up to seven 3. Consider the following statements related to
Agnilet engines, depending on the mission, e-YANTRA:
which are powered by Liquid Oxygen and I. It is a robotics outreach program funded
Kerosene. by the Ministry of Education and hosted at
 The rocket is also designed for launch from IIT Bombay.
more than 10 different launch ports. To II. Its objective is to provide hands-on
ensure its compatibility with multiple learning to engineering students who
launch ports, AgniKul has built a launch have limited access to labs and mentors.
pedestal named 'Dhanush' that will Which of the above given statement is/are
support the rocket's mobility across all its correct?
configurations. The Agnilet engine, which 1) I only 2) II only

AMIGOS IAS 2
3) Both I and II 4) Neither I nor II Technology (QT). Hence statement I is
Answer: 3 correct.
Explanation:  The new mission targets developing
 E-Yantra is a robotics-focused educational intermediate scale quantum computers
program hosted at the Indian Institute of with 50-1000 physical qubits in 8 years in
Technology (IIT) Bombay. It is funded by various platforms like superconducting and
the Ministry of Education and is part of the photonic technology. Satellite based
National Mission on Education through ICT secure quantum communications between
(NMEICT). Hence statement I is correct. ground stations over a range of 2000
 The program's goal is to use the talent of kilometres within India, long distance
young engineers to solve problems using secure quantum communications with
technology in a variety of areas. These other countries, inter-city quantum key
areas include: Agriculture, Manufacturing, distribution over 2000 km as well as multi-
Defense, Home, Smart-city maintenance, node Quantum network with quantum
Service industries. memories are also some of the deliverables
of the Mission. Hence statement II is
 Its objective is to provide hands-on learning
correct.
to engineering students who have limited
access to labs and mentors. It also enables  The mission will help develop
effective education on embedded systems magnetometers with high sensitivity in
and robotics. Hence statement II is correct. atomic systems and Atomic Clocks for
precision timing, communications and
4. Consider the following statements related to
navigation. It will also support design and
National Quantum Mission:
synthesis of quantum materials such as
I. It aims to seed, nurture and scale up superconductors, novel semiconductor
scientific and industrial R&D and create a structures and topological materials for
vibrant & innovative ecosystem in fabrication of quantum devices. Hence
Quantum Technology (QT). statement III is correct
II. It targets developing intermediate scale
 The mission would greatly benefit
quantum computers with 50-1000
communication, health, financial and
physical qubits in 8 years in various
energy sectors as well as drug design, and
platforms like superconducting and
space applications. It will provide a huge
photonic technology.
boost to National priorities like digital
III. This mission helps to develop India, Make in India, Skill India and Stand-
magnetometers with high sensitivity in up India, Start-up India, Self-reliant India
atomic systems and Atomic Clocks for and Sustainable Development Goals (SDG).
precision timing, communications and
5. Arctic Amplification, often seen in the news,
navigation.
refers to?
Which of the above given statements is/are
1) Warming differential between Arctic
correct?
region over Antarctic region.
1) I and II 2) II and III
2) warming differential between the poles
3) I and III 4) I, II, and III and the tropics.
Answer: 4 3) warming differential between the sub-
Explanation: tropics and the tropics.
 The Union Cabinet, chaired by the Hon'ble 4) Warming of Antarctic region abnormally.
Prime Minister Shri Narendra Modi, today Answer: 2
approved the National Quantum Mission Explanation:
(NQM) at a total cost of Rs.6003.65 crore
 World Metrological Organisation (WMO)
from 2023-24 to 2030-31, aiming to seed,
study highlighted that Arctic has warmed
nurture and scale up scientific and
twice as much as world average.
industrial R&D and create a vibrant &
innovative ecosystem in Quantum  About Artic Amplification:

AMIGOS IAS 2
 Refers to warming differential between the  The policy is aimed to set up high
poles and the tropics. Hence, option (2) is resolution topographical survey and
correct. mapping, with a high-accuracy Digital
 As per some other studies, Arctic has Elevation Model (DEM) by 2030.
warmed nearly 4 times faster than the rest  It aims to make India a World Leader in
of the world over 1979– 2021. Global Geospatial space with the best in
 Reasons for faster warming of Arctic region the class ecosystem for innovation.
 Polar amplification is much stronger in the  To develop a coherent national framework
Arctic than in Antarctica as it is an ocean in the country and leverage it to move
covered by sea ice, while Antarctica is an towards a digital economy and improve
elevated continent with permanent ice and services to citizens.
snow. Hence both the statements are correct.
 Melting of the ice cover in the Arctic 7. Consider the following statements with
exposes more land/water to the Sun, respect to “ANUSANDHAN NATIONAL
leading to more absorption of sunlight. RESEARCH FOUNDATION ACT, 2023”.
 Prevailing systems of air circulations I. The ANRF is to be implemented with a
transfer heat from the tropics to the poles. total budget of Rs. 50,000 crores for five
 Summer Arctic sea ice extent is shrinking by years (2023-28), out of which a major
12.6% per decade as a result of global share of around 70%, is estimated to come
warming. from non-government sources.
6. Consider the following statements with II. The ANRF aims to provide high level
respect to National Geospatial policy: strategic direction for research,
I. The policy is aimed to set up high innovation and entrepreneurship in the
resolution topographical survey and fields of natural sciences including
mapping, with a high-accuracy Digital mathematical sciences, engineering and
Elevation Model (DEM) by 2030. technology, environmental and earth
sciences, health and agriculture.
II. It is a citizen-centric policy based on Geo-
Spatial technology, which seeks Which of the above given statements is/are
to strengthen the Geospatial sector to correct?
support national development, economic 1) I only 2) II only
prosperity and a thriving information 3) Both I and II 4) Neither I nor II
economy. Answer: 3
Which of the above given statements is/are Explanation:
correct?  The ANRF aims to provide high level
1) I only 2) II only strategic direction for research, innovation
3) Both I and II 4) Neither I nor II and entrepreneurship in the fields of
Answer: 3 natural sciences including mathematical
Explanation: sciences, engineering and technology,
environmental and earth sciences, health
 In 2021, the Ministry of Science and
and agriculture. The ANRF is to be
Technology issued “Guidelines for
implemented with a total budget of Rs.
acquiring and producing Geospatial
50,000 crores for five years (2023-28), out
Data and Geospatial Data Services
of which a major share of around 70%, is
including Maps.
estimated to come from non-government
 It is a citizen-centric policy based on Geo- sources. ANRF will address many big
Spatial technology, which seeks challenges of present R&D ecosystem
to strengthen the Geospatial sector to including that of lower participation of
support national development, economic private sectors in R&D. Hence both the
prosperity and a thriving information statements are correct.
economy.

AMIGOS IAS 3
 The establishment of ANRF will prove to be • First announced during COP27 in Sharm el-
one of the most transformational steps in Sheikh (Egypt) in 2022. Hence, Statement I is
the country towards achieving global R&D correct.
leadership and making India Atmanirbhar. • Meant to compensate countries already
8. The 8th Mission Innovation (MI8) and dealing with climate change.
14th Clean Energy Ministerial (CEM14) on the • ✓ L&D refers to the negative consequences
sidelines of G20 Energy Transition Ministerial arising from the unavoidable risks of climate
Meeting (ETMM) hosted by India in: change, like rising sea levels, heatwaves, crop
1) Maharashtra 2) Delhi failures, etc.
3) Tamil Nadu 4) Goa • ✓ Warsaw International Mechanism (WIM) for
Answer: 4 L&D was established at COP19 in 2013 in
Explanation: Warsaw (Poland) to address L&D associated
 India hosted the joint Ministerial event of with climate change in developing countries.
8th Mission Innovation (MI8) and • Will be administered by the World Bank as a
14th Clean Energy Ministerial (CEM14) on World Bank-hosted Financial Intermediary
the sidelines of G20 Energy Transition Fund for an interim period of four years.
Ministerial Meeting (ETMM) on 19-22 July Hence, Statement III is correct.
2023 in Goa. • ✓ But managed by an independent
 The event was held on the sidelines of the secretariat. o Countries including UAE,
G20 Energy Transitions Ministerial Meeting Germany, UK, Japan, etc., pledged money to
and has a theme of "Advancing Clean the fund.
Energy Together". • Mobilization of $83.9 billion to climate finance
 The MI8's mission is to promote intelligent in the Green Climate Fund, Adaptation Fund,
cities. The MI Missions encourage Least Developed Countries Fund, and Special
international collaboration and Climate Change Fund.
coordination, allowing governments to • Declaration on a Global Climate Finance
work with the private sector to target Framework (India part of the declaration).
investment and action 10. Recently in news, the term ‘Tycho’ refers to
9. Consider the following statements with 1) Comet 2) Exoplanet
respect to Loss and Damage (L&D) Fund. 3) Meteoroid 4) Supernova
I. It was first announced during COP27 in Sharm Answer: 4
el-Sheikh (Egypt) in 2022. Explanation:
II. It was operationalized after COP28. Tycho is a Type Ia supernova, which
III. It will be administered by the World Bank as a occurs when a white dwarf star shreds its
World Bank-hosted Financial Intermediary companion star, triggering an explosion and
Fund for an interim period of four years. sending debris hurtling into space at
How many of the above statements is/are correct? tremendous speeds. It released as much
1) Only One 2) Two Only energy as the Sun would emit over ten billion
3) All Three 4) None years and blasted particles out into space near
Answer: 3 the speed of light. Hence, option 4 is correct.
Explanation: 11. Consider the following statements about the
• 28th COP28 of the United Nations Framework Lower Earth Orbit (LEO)
Convention on Climate Change (UNFCCC) was I. It is an orbit around the earth with an
held in Dubai, UAE. altitude between 160 to 2000 km.
• Loss and Damage (L&D) Fund was II. Unlike geosynchronous satellite, satellites
operationalised under it. Hence, Statement II in LEO have a small field of view and so can
is correct. observe and communicate with only a
fraction of the Earth at a time.

AMIGOS IAS 4
III. It has applications in the field of satellite I. It was formed from the amalgamation of
imaging and earth-observing satellites. Technical Development Establishment
Which of the above given statements is/are (TDEs) of the Indian Army and the
correct? Directorate of Technical Development &
1) I and II 2) II only Production (DTDP) with the Defence
3) I and III 4) I, II and III Science Organisation (DSO).
Answer: 4 II. Its vision is to empower the nation with
state-of-the-art indigenous defence
Explanation:
technologies and systems.
 The Lower Earth Orbit is an orbit around
Which of the above statements is/are correct?
the earth with an altitude between 160 –
2000 km. the satellites placed in LEO can 1) I only 2) II only
have a titled plane and the majority of 3) Both I and II 4) Neither I nor II
human space flights land in LEO. Hence, Answer: 3
statement I is correct. Explanation:
 Unlike geosynchronous satellite, satellites  DRDO was formed in 1958 from the
in LEO have a small field of view and so can amalgamation of the then already
observe and communicate with only a functioning Technical Development
fraction of the Earth at a time. That means Establishment (TDEs) of the Indian Army
that a network (or “constellation”) of and the Directorate of Technical
satellites is required to provide continuous Development & Production (DTDP) with
coverage. Satellites in lower regions of LEO the Defence Science Organisation (DSO).
also suffer from fast orbital decay and Hence statement I is correct.
require either periodic re-boosting to  Being a centre of excellence for the design
maintain a stable orbit or launching and development of state-of-the-art UAVs,
replacement satellites when old one’s re- Aero Gas Turbine Engine Technology,
enter. Hence, statement II is correct. Airborne Surveillance Systems,
 It has applications in the field of satellite Technologies and Systems related to
imaging, communication satellites, Parachute, Decelerators and Lighter-than-
remote-sensing satellites, and earth- Air systems. Hence statement II is correct.
observing satellites. The International  Making India a powerful defense system
Space Station is located in LEO. Hence, and move to develop in science and
statement III is correct. technology.
12. ‘BharOS’ recently seen in news is a/an 14. Consider the following statements with
1) Anti-virus 2) Chatbot reference to ‘Pinaka Rocket Launcher
3) Mobile operating system System’:
4) Positioning system I. It has been designed jointly by India and
Answer: 3 Russia
Explanation: II. It can fire 120 rockets in a span of 60
The BharOS is an operating system used in seconds
mobile phones. It was developed by J and K Which of the above given statements is/are
Operations Private Limited. IIT Madras was the correct?
pioneer in developing the app. There are No 1) I only 2) II only
Default Apps in the OS. Funded by the 3) Both I and II 4) Neither I nor II
Department of Science and Technology (DST), Answer: 4
the indigenous mobile operating system wants Explanation:
to cater to the approximately 100 crore mobile  Pinaka is a multiple rocket launcher
phone users in India. produced in India and developed by the
13. With reference to the Defence Research and Defence Research and Development
Development Organisation (DRDO), consider
the following statements:
AMIGOS IAS 5
Organisation (DRDO) for the Indian Army. microorganisms. Hence Statement II is
Hence statement I is incorrect. correct.
 New Pinaka ER gives it a range of around 70  If the chemical formulas do not change,
km, over the 45 km existing range of the then any changes are physical changes. An
missile. Pinaka can fire a salvo of 12 rockets example of physical is dissolving sugar in
over a period of 44 seconds. Hence water. Once the sugar has all dissolved, you
statement II is incorrect. have molecules of sugar mixed in with the
15. “Paris Agreement Synthesis Report” is given molecules of water. The chemical formulas
by at the end are the same as they were at the
1) UNEP 2) Climate Watch start, so this is a physical change. Hence
3) UNFCC 4) Green Peace Statement III is not correct.
Answer: 3 17. The colour of the sun appears reddish during
sunset. Which of the following is the reason
Explanation:
behind this phenomenon?
 Paris Agreement Synthesis Report:
1) Absorption of light rays by the
 It is given by UNFCC. atmosphere
 Implementing latest NDCs will increase 2) Scattering of light rays by atmospheric
emissions by approximately 8.8% by 2030, particles
relative to 2010 levels.
3) Human eyes are not sensitive to other
 40% of Parties outlined the roles of local colours during that part of the day
communities and rights of Indigenous 4) The sun emits different wavelengths of
Peoples in adaptation components of light during different times of the day.
NDCs.
Answer:2
16. Which of the following is/are a chemical
Explanation:
process?
 As sunlight travels through the earth’s
I. Melting of an ice cube
atmosphere, it gets scattered (changes its
II. Formation of vinegar from sugarcane juice
direction) by the atmospheric particles.
III. Mixing sugar in water Light of shorter wavelengths is scattered
Select the correct answer using the code given much more than light of longer
below. wavelengths. (The amount of scattering is
1) I only 2) II only inversely proportional to the fourth power
3) II and III only 4) I, II and III of the wavelength. This is known as
Answer:2 Rayleigh scattering). At sunset or sunrise,
 Explanation: the sun’s rays have to pass through a larger
distance in the atmosphere. Most of the
 Melting of ice is a physical change because
blue and other shorter wavelengths are
it involves change only in the physical state
removed by scattering. The least scattered
of water. Ice in solid state convert to water
light reaching our eyes; therefore, the sun
in the liquid state. Moreover, no new
looks reddish. This explains the reddish
chemical substance is formed in the
appearance of the sun and full moon near
process and this change is reversible.
the horizon.
Hence Statement I is not correct.
 This is also the reason that the sky appears
 Vinegar is an aqueous solution of acetic
blue during the day, as blue has a shorter
acid and trace chemicals that may include
wavelength than red and is scattered much
flavourings. Vinegar typically contains 5–
more strongly. In fact, violet gets scattered
20% by volume acetic acid. Usually, the
even more than blue, having a shorter
acetic acid is produced by the fermentation
wavelength. But since our eyes are more
of ethanol or sugars by acetic acid bacteria.
sensitive to blue than violet, we see the sky
Fermentation is the chemical breakdown of
blue.
a substance by bacteria, yeasts, or other

AMIGOS IAS 6
18. Consider the following statements related to Explanation:
semiconductors:  National Water Awards:
I. The resistivity of semiconductors is more  President of India conferred 4th National
than insulators but less than conductors. Water Awards, 2022.
II. The most commonly used semiconductor  Prominent winners are Madhya Pradesh
elements are Silicon and Germanium. (Best State), Ganjam in Odisha (Best
Which of the above given statements is/are District).
correct?  National Water Awards: Launched in 2018
1) I only 2) II only by Department of Water Resources, River
3) Both I and II 4) Neither I nor II Development & Ganga Rejuvenation.
Answer: 2  2nd and 3rd editions given for the years
Explanation: 2019 and 2020.
 Conductors, semiconductors, and  Awards not given in the year 2021 due to
insulators are classified on the basis of COVID pandemic.
conductivity/resistivity.  Objective: To encourage
 Conductors possess very low resistivity (or various stakeholders towards adopting a
high conductivity). holistic approach towards water resources
 Semiconductors have resistivity or management in the country.
conductivity intermediate to conductors 20. Consider the following statements with
and insulators. regard to the Human Genome Project:
 Insulators have high resistivity (or low I. It was aimed at identifying all the genes in
conductivity). human DNA.
 Thus, resistivity of semiconductors is more II. It can help in diagnosis, treatment and
than conductors but less than insulators. prevention of genetic disorders in human
Hence statement I is not correct. beings.
 Semiconductors can be: Which of the statements given above is/are
 Elemental semiconductors: Silicon (Si) and correct?
Germanium (Ge) 1) I only 2) II only
 Compound semiconductors: Examples are: 3) Both I and II 4) Neither I nor II
 Inorganic: CdS, GaAs, CdSe, InP, etc. Answer: 3
 Organic: anthracene, doped Explanation:
phthalocyanines, etc.  It was aimed at identifying all the genes
 Organic polymers: polypyrene, polyaniline, (approximately 20,000-25,000 genes) in
polythiophene, etc. human DNA, determine the complete
sequence of DNA bases in the human
 Most of the currently available
genome, create detailed genetic maps that
semiconductor devices are based on
would provide the location and order of
elemental semiconductors Si or Ge and
genes on each chromosome, develop three
compound inorganic semiconductors.
research tools that would allow scientists
Hence statement II is correct.
to identify genes involved in both rare and
19. Consider the following statements with common diseases. Hence statement I is
reference to National Water Awards 2022. correct.
I. Madhya Pradesh won Best state award.
 It can help in diagnosis, treatment and
II. Ganjam district in Odisha won Best District prevention of genetic disorders in human
Award. beings. Hence statement II is correct.
Select the correct answer:
1) Only I 2) Only II 21. Consider the following statements regarding
3) Both I and II 4) Neither I nor II CRISPR-Cas9:
Answer:3

AMIGOS IAS 7
I. Clustered Regularly Interspaced Short energy efficiency and better technology
Palindromic Repeats of genetic options with a 20-year time horizon.
information is used in some bacterial • During Hydro Chlorofluorocarbons Phase
species as part of an antiviral system. Out Management Plan (HPMP) Stage-II
II. Guide RNA (gRNA) binds to Cas9 and implementation, India completely phased
specifies the location at which Cas9 out the use of Hydro chlorofluorocarbon
will cut DNA. (HCFC)- 141b in manufacturing of rigid
Which of the statements given above is/are foam, the first among the developing
correct? countries to achieve the milestone.
1) I only 2) II only • As against target of 35% reduction from
3) Both I and II 4) Neither I nor II the baseline as on 1.1.2020, India
Answer: 3 achieved a reduction of 44%, highlighting
India’s efforts in protection of the
Explanation:
stratospheric ozone layer.
 CRISPR is a genetic code or sequence found
23. Bandicoot Robot is built by:
in bacteria. They Formed due to the activity
of the previous bacteriophages that had 1) HSC, Bangalore
infected them 2) Kerala based startup Gen Robotics
 Clustered Regularly Interspaced Short 3) New Delhi Municipal Corporation
Palindromic Repeats of genetic information 4) IIT Kanpur
is used in some bacterial species as part of Answer: 2
an antiviral system. Hence statement I is Explanation:
correct.  Bandicoot is India’s first ‘manhole cleaning
 Guide RNA (gRNA) binds to Cas9 and robot’ built by a Kerala-based startup Gen
specifies the location at which Cas9 will cut Robotics.
DNA. Hence statement II is correct.  It is an exoskeleton robot which cleans
22. Consider the following statements manholes without the need for humans to
I. India is the first country in the world to enter the pit.
develop a comprehensive Cooling Action  The robotic scavenger is equipped with
Plan. four robotic legs for smooth portability. It
II. India completely phased out the use of has a robotic arm with 360-degree motion
Hydro chlorofluorocarbon (HCFC)- 141b in which plucks out the solid waste from the
manufacturing of rigid foam. corners of the manhole and collects it in a
III. As of 2020, India achieved a reduction of bucket. It is also equipped with a water jet
44℅ of hydro chlorofluorocarbon which clears sewage blockages. There are
emissions against the target of 35℅. cameras attached to the machine which
Which of the above given statements is/are enables one to see the footage of the
correct? manhole from inside.
1) I only 2) I and II only 24. Consider the following statements with
3) II and III only 4) All the above reference to carbon nanotubes:
Answer: 4 I. These are cylindrical molecules consisting
of rolled-up sheets of single-layer carbon
Explanation:
atoms.
• India Cooling Action Plan: India is the first
II. They have low thermal conductivity as
country in the world to develop a
they are very elastic.
comprehensive Cooling Action Plan, which
seeks to provide an integrated vision Which of the above statements is/are correct?
towards cooling across sectors 1) I only 2) II only
encompassing inter alia reducing cooling 3) Both I and II 4) Neither I nor II
demand, refrigerant transition, enhancing Answer: 1
Explanation:

AMIGOS IAS 8
 Carbon Nanotubes are cylindrical proposals which includes online
molecules consisting of rolled-up sheets of submission of a new proposal,
single-layer carbon atoms. Hence editing/updating the details of proposals
statement I is correct. and displays status of the proposals at each
 Carbon Nanotubes are stronger than steel stage of the workflow.
wires and have high thermal and electrical  To enhance users, experience on PARIVESH
conductivity. They are very elastic and have with modern day web application, Ministry
very high tensile strength. Hence has expanded the scope of existing
statement II is incorrect. PARIVESH (2.0) leveraging on emerging
25. Which of the following public institutions are technology such as GIS, Advance Data
dedicated towards Robotics and Artificial Analytics, etc. for providing faster decisions
intelligence? on Green Clearances and robust
1) Centre for Artificial Intelligence and compliance monitoring by end-to-end
Robotics (CAIR), Bengaluru online appraisal and grant of clearances. An
2) Research and Development Establishment upgraded version of existing PARIVESH is
(Engineers), Pune supplemented with unique modules (Know
Your Approval, Know Your Customer,
3) Indian Space Research Organisation (ISRO)
Decision Support System etc).
4) Both (1) and (2)
 Major modules in PARIVESH 2.0 viz: end to
Answer: 4
end online processing of Category A and B
Explanation: environmental clearance proposals was
Centre for Artificial Intelligence and Robotics developed and rolled out at Central and
(CAIR), Bengaluru and Research and SEIAA level, respectively. In addition, all
Development Establishment (Engineers) major functionalities of other major
{R&DE(E)}, Pune are prime laboratories of clearances (FC/WL&CRZ) have been
Defence Research and Development developed and rolled out. In CRZ clearance,
Organisation (DRDO) working in the area of all nine State Coastal Zonal Management
artificial intelligence and robotics. Authority were onboarded, for the first
26. PARIVESH, online application sometimes time, on PARIVESH 2.0 for online
seen in the news is related to submission and processing of applications.
1) Tracking of learning levels for school going Furthermore, PARIVESH 2.0 is also
children integrated with Gatishakti and National
2) To encourage students to participate in Single window portal.
research and development 27. Consider the following statements regarding
3) online submission and monitoring of the COP 28 of the UN Climate Change Conference
proposals submitted by the proponents I. It was held in Egypt.
for seeking Environment, Forest, Wildlife II. The major outcome from COP 28 is the
and CRZ Clearances decision on Outcome of the First Global
4) Create awareness on use of electric Stocktake.
vehicles Which of the above given statements is/are
Answer:3 correct?
Explanation: 1) I only 2) II only
 PARIVESH is a web based, role-based 3) Both I and II 4) Neither I nor II
workflow application which has been Answer: 2
developed for online submission and Explanation:
monitoring of the proposals submitted by  28th Session of the UN Climate Change
the proponents for seeking Environment, Conference (COP 28):
Forest, Wildlife and CRZ Clearances from
 An inter-ministerial delegation from India
Central, State and district level authorities.
attended the 28th Session of the UN
It automates the entire tracking of
Climate Change Conference (COP 28) held

AMIGOS IAS 9
in Dubai, United Arab Emirates from 30th 1) 1 only 2) 2 only
November’2023 to 13th December’2023. 3) Both 1 and 2 4) Neither 1 nor 2
 The major outcome from COP 28 included Answer:1
the decision on Outcome of the First Global Explanation:
Stocktake, ratcheting up global climate  India’s achievements against the NDC
ambition before the end of the decade and targets:
agreement on the operationalization of loss
 As per India’s first Nationally Determined
and damage fund. These global efforts will be
Contribution (NDC) submitted in the year
taken up by the countries in a nationally
2015, India had the target of:
determined manner taking into account.
 Reducing the emissions intensity of its GDP
28. Match the following pairs with reference to
by 33 to 35 percent by 2030 from 2005
Robots and their applications:
level; and
I. Ai-Da A) India’s first indigenous
 Achieving about 40 percent cumulative
industrial robot
electric power installed capacity from non-
II. Groundhog B) Anthropomorphic fossil fuel-based energy resources by 2030.
humanoid robot  These two targets have been achieved well
III. ATLAS C) Mine-Mapping Robot ahead of the time. As on 31st October,
I.V. BRABO D) First robot artist to 2023; the cumulative electric power
stage an exhibition installed capacity from non-fossil fuel-
Options: based energy resources is 186.46 MW,
1) I-A II-B III-C IV-D which is the 43.81% of the total cumulative
2) I-B II-D III-A IV-C electric power installed capacity. The
3) I-D II-C III-B IV-A emission intensity of its GDP has been
reduced by 33 percent between 2005 and
4) I-C II-D III-A IV-B
2019.
Answer: 3
 In August 2022, India updated its NDC
Explanation:
according to which target to reduce
 Ai-Da is a humanoid robot artist that uses emissions intensity of its GDP has been
AI to draw, paint, and perform. enhanced to 45 percent by 2030 from 2005
 Groundhog is an autonomous robotic level, and the target on cumulative electric
system developed by Carnegie Mellon power installed capacity from non-fossil
University in 2010 to explore and map fuel-based energy resources has been
abandoned mines enhanced to 50% by 2030.
 Atlas is a bipedal humanoid robot 30. Consider the following statements with
developed by Boston Dynamics for search reference to the immersive technologies:
and rescue tasks. I. Virtual reality immerses users in a fully
 BRABO stands for "Bravo Robot" and artificial digital environment.
is India's first industrial robot. It's designed II. In Augmented Reality, physical and digital
to be developed for micro, small, and objects co-exist and interact in real time.
medium enterprises (MSME) in India Which of the above statements is/are
29. Consider the following statements incorrect?
1) As of October, 2023, the cumulative 1) I only 2) II only
electric power installed capacity from 3) Both I and II 4) Neither I nor II
non-fossil fuel-based energy resources is Answer: 2
more than 200MW in India.
Explanation:
2) The emission intensity of its GDP has been
 Virtual reality immerses users in a fully
reduced by 33 percent between 2005 and
artificial digital environment. Hence
2019.
statement I is correct.
Which of the above given statements is/are
incorrect?
AMIGOS IAS 10
 Mixed Reality is an advanced form of Limited (BBNL) to use the dark fibre laid by
augmented reality where physical and the latter.
digital objects co-exist and interact in real  The hitherto unused infrastructure will be
time. Hence statement II is incorrect. used by these companies to expand their
 Augmented Reality (AR) is a live direct view presence in rural India and offer 4G as well
of a physical, real-world environment as other long-term evolution (LTE) mobile
whose elements are augmented (or telephony services.
supplemented) by computer-generated 3D  The bandwidth requested by all the
models, animations, videos etc. The telecom service providers is a part of the
technology enhances user’s current dark fibre laid out by BBNL while working
perception of reality. Augmentation is in on the BharatNet project.
real time and the information is overlaid on 32. Which of the following diseases are caused
the live view of the device’s camera. by viruses?
Augmented Reality takes our current I. Hepatitis C
reality and adds overlays of various types of
II. Chicken Pox
information on it. In other words, it simply
III. Common Cold
augments our existing reality. AR usually
takes place on your smartphone or tablet. IV. Malaria
 Perhaps the most recent development in Select the correct answer using the code given
immersive or reality technologies, mixed below:
reality can be considered as an advanced 1) I, II and III only
form of AR, where overlays of information 2) II and IV only
are placed on existing reality but shown 3) I, III and IV only
through your mixed reality glasses or 4) II, III and IV only
headsets. The physical and digital objects Answer: 1
co-exist and interact in real-time. Explanation:
31. Dark fibre is a  Hepatitis C is a viral infection that can
1) It is a cable laid on the sea bed between cause both short-term and long-term
land-based stations to carry illness. It is caused by the Hepatitis C virus
telecommunication signals across (HCV).
stretches of ocean and sea.  Chickenpox is a highly contagious disease
2) It is part of the internet that isn't visible to caused by the varicella-zoster virus
search engines and requires the use of an (VZV). It's a member of the herpesvirus
anonymizing browser. family and can be spread from 1–2 days
3) It is the collective name for a number of before blisters appear until all the blisters
malicious software variants. have crusted over
4) It is the extra optical fibre laid by  The common cold is an upper respiratory
companies in order to avoid cost infection that affects your nose, throat,
repetition when more bandwidth is sinuses, and windpipe. It can be caused by
needed many different viruses, but rhinoviruses
Answer: 4 are the most commonly implicated.
Explanation:  Malaria is caused by a parasite called
 Dark fibre is the extra optical fibre laid by Plasmodium, which is spread to humans
companies in order to avoid cost repetition through the bites of infected mosquitoes.
when more bandwidth is needed. Hence IV is incorrect.
 In a bid to cut down on capital expenditure, 33. Consider the following statements regarding
the three major telecom service providers Green Credit Programme
– Reliance Jio Infocom, Bharti Airtel and I. It was launched on the side-lines of COP28
Vodafone Idea — have reached out to OF UNFCCC.
state-run Bharat Broadband Network

AMIGOS IAS 11
II. Indian Council of Forestry Research and 34. Consider the following statements:
Education (ICFRE) is designated as I. Chromosomal disorders are caused by
administrator and is responsible for absence of one or more chromosomes.
implementation and management of GCP. II. Down’s syndrome and Turner’s syndrome
Which of the above given statements is/are are examples of chromosomal disorders.
correct? Which of the statements given above is/are
1) I only 2) II only correct?
3) Both I and II 4) Neither I nor II 1) I only 2) II only
Answer: 3 3) Both I and II 4) Neither I nor II
Explanation: Answer: 3
Green Credit Programme (GCP):  Explanation:
 Green Credit Initiative was launched by Chromosomal disorders are caused due to
Hon’ble Prime Minister on the side-lines of absence or excess or abnormal
COP 28. It is an initiative within the arrangement of one or more
governments Lifestyle for Environment or chromosomes.
LIFE movement. The Green Credit Rules,  Down’s syndrome, Turner’s syndrome,
2023, has been notified on 12th October Klinefelter’s syndrome are common
2023 under the Environment Protection examples of chromosomal disorders.
Act 1986. These rules put in place a Hence both statements are correct.
mechanism to encourage voluntary 35. Consider the following statements regarding
environmental positive actions resulting in ‘BiodiverCities by 2030’:
issuance of green credits. In its initial
I. It aims to support city governments,
phase, voluntary tree plantation is
businesses and citizens, to enable cities to
envisaged on degraded land, waste land,
live in harmony with nature by 2030
watershed area etc. under the control and
management of Forest departments. II. It is released by World Economic Forum in
collaboration with United Nations
 The generation of Green Credit under
Development Programme
Green Credit Rules, 2023 is independent of
III. The programme employs Artificial
the carbon credit under Carbon Credit
Intelligence to address challenges related
Trading Scheme 2023.
to nature and cities
 Governance structure of GCP includes the
Which of the above given statements is/are
steering committee members from
correct?
concerned ministries/departments,
experts and institutions. Indian Council of 1) I only 2) I and II only
Forestry Research and Education (ICFRE) is 3) II and III only 4) I and III only
designated as GCP administrator and is Answer: 4
responsible for implementation and Explanation:
management of GCP. Digital process of GCP  It aims to support city governments,
includes dedicated web platform and GC businesses and citizens, to enable cities to
registry for streamlining the operations. In live in harmony with nature by 2030. The
addition to these methodologies and initiative brings together multidisciplinary
guidelines, including registration, expertise, combines existing initiatives and
accounting and GC issuance monitoring surfaces innovative solutions to promote
ensures the transparency and sustainable, inclusive and nature-positive
accountability of GCP. urban development at a global scale.
Hence statement I is correct.
 The BiodiverCities by 2030 initiative is
convened in collaboration with the World
Economic Forum and the Government of
Colombia. Hence statement II is incorrect.

AMIGOS IAS 12
 The initiative uses artificial intelligence and which has been installed under the
crowdsourcing technologies through the government’s National Supercomputing
Forum’s Strategic Intelligence and Mission. Hence statement III is correct.
Uplink platforms, collecting up-to-date 37. Which of the following pair is incorrect
online content and building a community regarding the blue flag certified beaches and
of innovators and entrepreneurs to their respective states?
address the world’s most pressing 1) Shivrajpur beach - Maharashtra
challenges on nature and cities. Hence 2) Kappad beach - Kerala
statement III is correct. 3) Kovalam beach - Tamil Nadu
36. Consider the following statements about the 4) Eden beach - Puducherry
National Supercomputing Mission
Answer: 1
I. It aims to indigenise the development and
Explanation:
manufacturing of powerful computers.
 Beaches in urban India are widely
II. It is implemented by the National
recognized as popular tourist destinations
Informatics Centre (NIC).
all year round. The coastal areas of
III. Under this mission, the
Visakhapatnam, Mumbai, Chennai, Goa,
supercomputer ‘Param Pravega’ has been
Kerala, Odisha have gained significant
installed.
attention from tourists worldwide, leading
Which of the above given statements is/are to a substantial influx of visitors. The
correct? presence of these pristine beaches not only
1) I and II only 2) I, II and III serves as a source of income for many
3) III only 4) I and III coastal communities but also plays a crucial
Answer: 4 role in boosting the local economy. The
Explanation: beachline across different regions is
 The National Supercomputing Mission aims encountering a range of issues such as
to indigenise the development and pollution, overcrowding due to high tourist
manufacturing of powerful computers. influx, and inadequate upkeep. The
presence of marine litter possesses a
 It was launched in 2015 to enhance the
significant threat to the marine and coastal
research capacities and capabilities in the
ecosystem thus hampering tourism.
country by connecting them to form a
Citizens and Urban Local Bodies (ULBs) are
Supercomputing grid, with National
growing increasingly conscious of this
Knowledge Network (NKN) as the
threat. Joggers and runners across the
backbone.
country are embracing the practice of
 The NKN project is aimed at establishing a
plogging on beaches, which entails picking
strong and robust Indian network that will
up litter while jogging or running. This
be capable of providing secure and reliable
activity not only helps in keeping the
connectivity. Hence statement I is correct.
beaches clean but also promotes a healthy
 The Mission is being jointly steered by lifestyle.
the Department of Science and Technology
 Efforts are being made by the local
(DST) and the Ministry of Electronics and
authorities and community groups to enhance
Information Technology (MeitY).
waste management and sanitation on the
 It is implemented by the Centre for beaches. Regular beach cleanup drives and
Development of Advanced Computing (C- restoration activities are being carried out to
DAC), Pune, and the IISc, Bengaluru. Hence maintain the cleanliness of the beaches.
statement II is incorrect. Additionally, the ULBs are organizing various
 The Indian Institute of Science (IISc) awareness programs to educate people about
Bengaluru installed the the significance of preserving natural
supercomputer ‘Param Pravega’ with resources and marine life. These initiatives are
a supercomputing capacity of 3.3 petaflops being undertaken as part of the Swachh

AMIGOS IAS 13
Bharat Mission Urban, which aims to promote III. Its main purpose is to work as a high-
cleanliness and hygiene in urban areas. power and high-capacity reservoir for
Following the protocols for clean, safe and excess wind and solar energy.
environment-friendly beaches, marinas and Which of the above given statements is/are
sustainable boating tourism operators, India correct?
has 12 Blue Flag Beaches namely, Odisha’s 1) I and II 2) I and III
Golden Beach, Gujarat’s Shivrajpur Beach, 3) I, II and III 4) II and III
Kappad Beach of Kerala, Diu’s Ghoghla Beach,
Answer: 3
Radhanagar Beach in Andaman & Nicobar,
Explanation:
Karnataka’a Kasadgod & Padubidri beaches,
Rushikonda Beach of Andhra Pradesh, TN’s  A “sand battery” is a high temperature
Kovalam Beach, Eden Beach of Puducherry, thermal energy storage that uses sand or
Lakshwadeep’s Minicoy Thundi & Kadmat sand-like materials as its storage medium.
beaches. It stores energy in sand as heat. Hence
38. Which of the following statements best statement I and II are correct.
describes “Virtual Private Network”?  Sand is a very effective medium for
1) It is a private computer network of an retaining heat over a long period, storing
organization where the remote users can power for months at a time.
transmit encrypted information through  Its main purpose is to work as a high-power
the server of the organization and high-capacity reservoir for excess wind
2) It is a computer network across a public and solar energy. The energy is stored as
internet that provides users access to their heat, which can be used to heat homes, or
organization’s network while maintaining to provide hot steam and high temperature
the security of the information process heat to industries that are often
transmitted fossil-fuel dependent. Hence statement III
is correct.
3) It is a computer network in which users
can access a shared pool of computing  The sand battery helps to ambitiously
resources through a service provider. upscale renewables production by
4) None of the above statements. ensuring there’s always a way to benefit
from clean energy, even if the surplus is
Answer: 2
massive.
Explanation:
40. Consider the following statements with
 A Virtual Private Network (VPN) is a service regard to Government Instant Messaging
that creates a secure connection between System (GIMS):
a user's computer and the internet. It
I. It is designed and developed by National
provides a private tunnel for data, making
Informatics Centre.
the user anonymous and blocking anyone
II. It is considered safer than the other
from tracking their movements.
private popular messaging platforms.
 A VPN hides a user's IP address by
Which of the statements given above is/are
redirecting it through a remote server run
correct?
by a VPN host. This allows remote workers
to connect to their company's internet or 1) I only 2) II only
restricted applications 3) Both I and II 4) Neither I nor II
39. Consider the following statements about Answer: 3
Sand Battery Explanation:
I. A sand battery is a high temperature  Government Instant Messaging System
thermal energy storage that uses sand or (GIMS) is designed and developed by
sand-like materials as its storage medium. National Informatics Centre (NIC). It is
II. It stores energy in sand as heat. meant to be used by Central and State
government departments and
organisations for intra and inter

AMIGOS IAS 14
organisation communications. Hence 2) The Terai region of Bihar
statement I is correct. 3) Indo–Myanmar border region of Arunachal
 This platform is considered safer than the Pradesh
popular messaging platforms like 4) Eastern Ghats
WhatsApp, as the server hosting it is Answer: 1
installed within the country and the Explanation:
information stored would be on
 Red-rumped Hawklet Scientific name –
government-based cloud, i.e. NIC data
Epithemis wayanadensis
centres. Hence statement II is correct.
 It is named to mark the lush and biodiverse
41. Consider the following statements with
Wayanad plateau in the Western Ghats.
reference to Radio-frequency identification
(RFID) technology:  Habitat - Shaded streamside marsh near
Wayanad’s forested terrain and parts of
I. RFID uses radio waves to communicate
the Nilgiri Coorg landscapes
between two objects.
 Features
II. RFID Tag needs to be within the line of
sight of the reader to work.  Darker pigmentation A restricted red
III. RFID can identify the vehicle by reading coloration on the abdomen Absence of the
the registration number of vehicles. yellow ante humeral stripe
Which of the statements given above is/are  Growing conditions – It is profoundly
correct? seasonal which is observed in during the
month of October in skies. It spends the
1) I only 2) II and III only
remainder of the year in the aquatic larval
3) I and II only 4) I, II and III
stage. It bore a striking resemblance to
Answer: 1 Ruby-tailed Hawklet.
Explanation:  Significance– A 2% genetic variance
 Radio Frequency Identification (RFID) is a indicates the presence of a new species. In
type of passive wireless technology that this case, an astonishing 12% genetic
allows for tracking or matching of an item difference was observed. This marks the
or individual. RFID tag uses radio waves to 1st instance of an Indian dragonfly being
communicate its identity and other documented with genetic evidence
information. Hence statement I is correct. substantiating morphological distinctions
 RFID tag does not need to be very close to 43. In the context of cyber security, consider the
the reader or in the line of sight to work. A following statements:
tag can be read from up to several feet I. The content on Dark Net is encrypted and
away and does not need to be within direct requires specific browser for net surfing.
line-of-sight of the reader to be tracked. II. All activities associated with the deep web
One just simply has to be within a reading are nefarious.
distance from the scanner. Hence
III. ‘Deep web’ is part of greater ‘dark net’
statement II is incorrect
also known as ‘dark web’.
 RFID does not directly read the registration
Which of the statements given above is/are
number of vehicles. In fact, RFID
incorrect?
technology uses an Electronic Produce
1) I only 2) I and II only
Code (EPC) through which every vehicle
can be uniquely identified. This code is 3) I and III only 4) II and III only
different from the vehicle’s registration Answer: 4
number and exclusive to it on a global Explanation:
scale. Hence statement III is incorrect.  The content on Dark Net is encrypted and
42. Red-rumped Hawklet, a new dragonfly requires specific browser for net surfing.
species has been discovered in which of the Hence statement I is correct.
following region?  Not all activities associated with the deep
1) Western Ghats web are nefarious. In most cases, these
AMIGOS IAS 15
pages are not searchable through 1) X rays
traditional channels because they are 2) Ultraviolet radiation
password-protected and require specific 3) Gamma rays
tools or authorization (like a log-in) in order 4) Infrared radiation
to access them. Hence statement II is
Answer: 4
incorrect.
Explanation:
 The ‘dark net’ also known as the ‘dark web’
 Thermal imaging uses infrared (IR)
is part of the greater ‘deep web’, a network
radiation to create visible images that
of secret websites that exist on an
show the temperature differences in a
encrypted network. Hence statement III is
scene
incorrect.
44. Consider the following statements about  A thermal imager (also known as a thermal
iDEX camera) is essentially a heat sensor that is
capable of detecting tiny differences in
I. It aims to support projects requiring
temperature.
support beyond Rs. 10 crores up to Rs. 100
crores.  The device collects the infrared radiation
from objects in the scene and creates an
II. It is an ecosystem to foster innovation &
electronic image based on information
technology development in defence and
about the temperature differences.
aerospace by engaging innovators &
entrepreneurs.  Because objects are rarely precisely the
same temperature as other objects around
III. It provides funding/grants to Micro Small
them, a thermal camera can detect them
and Medium Enterprises (MSMEs), start-
and they will appear as distinct in a thermal
ups, individual innovators, R&D institutes,
image.
and academia.
46. Ecomark Scheme, recently seen in news is
Which of the above statements is/are correct?
implemented by which of the following
1) I and II 2) II and III
ministry?
3) I and III 4) I, II and III
1) Ministry of Environment, Forest and
Answer: 2 Climate Change
Explanation: 2) Ministry of New and Renewable Energy
 iDEX aims to support projects requiring 3) Ministry of Micro, Small and Medium
support beyond Rs. 1.5 crores up to Rs. 10 Enterprises
crores. Hence statement I is incorrect.
4) Ministry of Tribal Affairs
 iDEX was launched to provide wider Answer: 1
publicity and better visibility of iDEX
 Explanation:
activities and enable more efficient running
To take ahead the ‘LiFE’ (Lifestyle for
of future challenges through better
Environment’ movement), the Ministry of
information management.
Environment, Forest and Climate
 It is an ecosystem to foster innovation & Change has introduced the Ecomark
technology development in defence and Scheme.
aerospace by engaging innovators &
 Ecomark Scheme promotes Eco-Friendly
entrepreneurs. Hence statement II is
Products. It provides accreditation and
correct.
labelling for household and consumer
 It provides funding/grants to Micro Small products that meet specific environmental
and Medium Enterprises (MSMEs), start- criteria while maintaining quality standards
ups, individual innovators, R&D institutes, as per Indian norms.
and academia. Hence statement III is
 The Central Pollution Control Board
correct.
administers the Eco mark Scheme in
45. Which of the following rays or radiations are
partnership with Bureau of Indian
used in thermal imaging?
Standards (BIS). Products accredited under

AMIGOS IAS 16
the Ecomark Scheme will adhere to specific IV. Agni is an intermediate-range surface-to-
environmental criteria, ensuring minimal surface ballistic missile.
environmental impact. Which of the above given statements is/are
 It will build consumer awareness of correct?
environmental issues and encourage eco- 1) I, II and III 2) II, III and IV
conscious choices. It will also motivate 3) I, II and IV 4) I, III and IV
manufacturers to shift towards Answer: 4
environmentally friendly production. Explanation:
47. Consider the following statements
 The Integrated Guided Missile
 Israel has become a major supplier of Development Programme (IGMDP)
defence equipment to India, standing brought together the country’s scientific
second to Russia. community, academic institutions, R&D
 Barak 8 a long-range surface to air missile laboratories, industries and the three
was jointly developed by India’s Defence defence services in giving shape to the
Research & Development Organisation strategic, indigenous missile systems. It
(DRDO) and Israel Aerospace Industries formally got the approval of the Indian
(IAI). government on July 26, 1983. It was the
Choose the correct code: brainchild of renowned scientist Dr APJ
1) I only 2) II only Abdul Kalam and was intended to attain
3) Both I and II 4) Neither I nor II self-sufficiency in the field of missile
Answer: 3 technology.
Explanation:  The missiles developed under IGMDP are –
 Israel has become a major supplier of  Prithvi is a short-range surface-to-surface
defence equipment to India, standing ballistic missile. Hence statement I is
second to Russia on a few occasions. As per correct.
the data furnished by the Indian Ministry of  Trishul is a short-range low-level surface-
Defence (MoD) Israel was ahead of Russia to-air missile. Hence statement II is
in the year 2013–2014 and 2015–2016, in incorrect.
terms of signing defence contracts with  Akash is a medium-range surface-to-air
India. Hence statement I is correct. missile. Hence statement III is correct.
 Barak 8 was jointly developed by India’s  Agni is an intermediate-range surface-to-
Defence Research & Development surface ballistic missile. Hence statement
Organisation (DRDO) and Israel Aerospace IV is correct.
Industries (IAI). The Barak 8 missile defence 49. Consider the following statements
system is produced by Israel’s Directorate I. The Defence Research and Development
of Research and Development (DDR&D), Organisation (DRDO) was established in
Elta Systems, Rafael Advanced Defence 1958.
Systems and India’s Bharat Dynamics II. The Indian Space Research Organisation
limited (BDL). Hence statement II is (ISRO) was established in 1969.
correct.
III. The Department of Space (DoS) was
48. Consider the following statements regarding established in 1972.
the Integrated Guided Missile Development
Which of the above given statements is/are
Programme (IGMDP)
correct?
I. Prithvi is a short-range surface-to-surface
1) I and II 2) II and III
ballistic missile.
3) I and III 4) I, II and III
II. Trishul is a long-range surface-to-air
missile. Answer: 4
III. Akash is a medium-range surface-to-air Explanation:
missile  DRDO is the Research and Development
wing of the Ministry of Defence with a

AMIGOS IAS 17
vision to empower India with cutting-edge connects a wide range of key actors
defence technologies. DRDO was formed towards a rapid global transition to
in 1958 from the amalgamation of the efficient and climate-friendly cooling.
then-already functioning Technical  The key actors include government, cities,
Development Establishment (TDEs) of the international organizations, businesses,
Indian Army and the Directorate of finance, academia and civil society groups.
Technical Development & Production Hence statement I is correct.
(DTDP) with the Defence Science  The Cool Coalition is led by: UN
Organisation (DSO). Hence statement I is Environment, Climate and Clean Air
correct. Coalition, Kigali Cooling Efficiency Program
 The Indian Space Research Organisation and Sustainable Energy for All (SEforALL).
(ISRO) was established in 1969. ISRO is the Hence statement II is incorrect.
space agency under the Department of  It is a unified initiative that links action
Space of the Government of India, across the Kigali Amendment, Paris
headquartered in the city of Bengaluru, Agreement and Sustainable Development
Karnataka. ISRO’s vision is to harness space Goals. The 3 pillars of the cool coalition
technology for national development while include: Advocacy, Action, Knowledge
pursuing space science research and Exchange. Hence statement III is correct.
planetary exploration. Hence statement II
51. Consider the following statements Mission
is correct.
Shakti Anti Satellite Weapons Test
 The Department of Space was established I. It was a joint collaboration of the DRDO
in 1972. Its primary objective of promoting and ISRO.
the development and application of space
II. It struck a predetermined target orbiting
science and technology to assist in the all-
at a distance of 30000 km from the Earth’s
around development of the nation. Hence
surface.
statement III is correct.
III. India became only the fourth country to
50. Consider the following statements about
carry out an anti-satellite missile test.
U.N. Environment Programme's (UNEP) Cool
Coalition Which of the above given statements is/are
correct?
I. It is a global multi-stakeholder network
that aims to inspire ambition and 1) I and II 2) II and III
accelerate action on the transition to 3) I and III 4) I, II and III
clean and efficient cooling. Answer:3
II. It is led by UN Environment, IUNC and Explanation:
World Economic Forum.  Mission Shakti is a joint programme of the
III. It is a unified initiative that links action Defence Research and Development
across the Kigali Amendment, Paris Organisation (DRDO) and the Indian Space
Agreement and Sustainable Development Research Organisation (ISRO). Hence
Goals. statement I is correct.
Which of the above given statements is/are  It struck a predetermined target which was
correct? a redundant Indian satellite that was
1) I and II 2) II and III orbiting at a distance of 300 km from the
3) I and III 4) I, II and III Earth’s surface. As per official sources, the
satellite that had been knocked out was
Answer: 3
Microsat R, a micro-satellite launched by
Explanation:
ISRO in January, 2019. Hence statement II
 Cool Coalition is dedicated to inspiring is incorrect.
ambition, mobilizing action, and
 The first anti-satellite test (ASAT) was
accelerating progress toward UN 2019
carried out by the US military way back in
Climate Change Summit. The Cool Coalition
1959. The then Soviet Union followed a
is a global multi-stakeholder network that
year later. Thereafter, the two countries
AMIGOS IAS 18
carried out a series of such tests up till early 53. The function of control rods in a nuclear
1980s.After that there was a lull, broken reactor is to
only by the Chinese test in 2007.India 1) Absorb neutrons
became only the fourth country to carry 2) Slow down the speed of neutrons
out an anti-satellite missile test. Hence 3) Cool down the reactor
statement III is correct.
4) Increase the speed of neutrons
52. Consider the following statements about
Answer: 1
Bhoj wetland
Explanation:
I. It consists of two contiguous human-made
reservoirs located in the city of Bhopal,  Control rods are an essential safety feature
Madhya Pradesh. in nuclear reactors. They are made of a
material that can absorb neutrons, such as
II. Recently Keoladeo National Park and Bhoj
boron, cadmium, or hafnium. When
wetland were added under the Montreux
inserted into the reactor core, they can
record, maintained under the Ramsar
help regulate the nuclear reaction by
convention.
absorbing excess neutrons and slowing
Which of the above given statements is/are down the rate of fission.
correct?
 By absorbing neutrons, control rods can
1) I and II 2) II only
help prevent the reactor from overheating
3) I only 4) Neither I nor II and potentially causing a nuclear
Answer: 3 meltdown. They can also be used to shut
Explanation: down the reactor entirely in case of an
 It consists of two contiguous human-made emergency. Hence option 1 is correct.
reservoirs, the upper and lower lakes 54. ‘Pterygotrigla intermedica’ a species recently
located in the city of Bhopal, Madhya seen in news are
Pradesh. The Upper Lake is called Bhojtal 1) Wasps 2) Bats
(Bada Talaab) and Lower Lake is known as 3) frogs 4) Fishes
Chhota Talaab. Answer: 4
 The lakes are very rich in biodiversity, Explanation:
particularly for macrophytes,
‘Pterygotrigla intermedica’ commonly known
phytoplankton and zooplankton. It has
as gurnards or sea-robins and belongs to the
more than 15 kinds of fishes and several
family Triglidae. The specimens of this fish
vulnerable species including turtles,
were found to be very distinct from other
amphibians and aquatic invertebrates. It is
gurnad species in various aspects, such as
designated as the Ramsar site in 2002.
snout length, shape of the internuchal space,
Hence statement I is correct.
and size of the cleithral spine. It is the fourth
 The Montreux record is a register of species of Pterygotrigla genus reported in
wetland sides on the list of wetlands of India so far, and there are a total of 178
international importance where changes in species of the Triglidae family worldwide.
ecological character have occurred, are 55. Consider the following statements about
occurring, or are likely to occur as a result Nuclear Non-Proliferation Treaty (NPT)
of technological developments, pollution
I. It defines Nuclear-weapon states as those
or other human interference.
that manufactured and exploded a
 Currently, 48 sites were listed in the nuclear weapon before 1st January 1967.
Montreux Record. Indian sites in the like
II. All the members of the United Nations are
Keoladeo National Park- Rajasthan, Loktak
members of this treaty.
Lake – Manipur are included in Montreux
Which of the above given statements is/are
Record. Hence statement II is incorrect.
correct?
1) I only 2) II only
3) Both I and II 4) Neither I nor II

AMIGOS IAS 19
Answer: 1 Ltd. and Bharat Electronics Limited are its
Explanation: production partners.
 The Nuclear Non-Proliferation Treaty is a  TAPAS BH-201 is a medium-altitude long-
multilateral treaty aimed at limiting the endurance unmanned aerial vehicle
spread of nuclear weapons including three  It is being designed to acquire real-time,
elements: high-quality pictures and signal intelligence
 non-proliferation from fields of concern at medium to long
 disarmament and ranges.
 peaceful use of nuclear energy.  It is being developed to carry out
surveillance and reconnaissance (ISR) roles
 According to this treaty, Nuclear-weapon
and is capable of carrying different
states are those that manufactured and
combinations of advanced payloads and
exploded a nuclear weapon before
capable of auto-landing among others
1st January 1967.
 This scheme will help in mapping with
 It requires countries to give up any present
Improvised Technology in Village Areas
or plans to build nuclear weapons in return
with the help of drones to map properties
for access to peaceful uses of nuclear
in villages. People in rural areas now have
energy.
clear evidence of ownership.
 It represents the only binding commitment
57. This scheme will help in mapping with
in a multilateral treaty to the goal of
Improvised Technology in Village Areas with
disarmament by the nuclear-weapon
the help of drones to map properties in
States. Hence statement I is correct.
villages. People in rural areas now have clear
 The treaty was signed in 1968 and entered evidence of ownership.
into force in 1970. Presently, it has 191
Which one of the following schemes is being
member states.
talked about in the para given above?
 Four UN member states have never
1) NATMO
accepted the NPT – India, Israel, Pakistan,
2) Mission Drone Shakti
and South Sudan. Hence statement II is
incorrect. 3) National Atlas
56. The term ‘Tapas-BH’ was recently seen in 4) SWAMITVA
news, corresponds to which of the following? Answer: 4
1) It will acquire real-time, high-quality Explanation:
pictures and signal intelligence from fields SWAMITVA- Drones are being used as part of
of concern at Short to Medium ranges. a plan called Survey of Villages and Mapping
2) It is developed by the Defence Research with Improvised Technology in Village Areas to
and Development Organisation (DRDO). map properties in villages. Rural residents now
3) It is high altitude, long endurance (HALE) have undeniable proof of possession.
Unmanned Aerial Vehicle (UAV). SVAMITVA Scheme is a reformative step
towards establishment of clear ownership of
4) It is capable of carrying different
property in rural inhabited (Abadi) areas
combinations of advanced payload and
capable of auto-landing among others. 58. Consider the following statements with
respect to the Environmental DNA (eDNA)
Answer: 4
I. It is the genetic material obtained from
Explanation:
the fossils and dead species.
 Rustom-2 also goes by the name Tapas-BH.
II. It originates from cellular material shed by
(Tactical Airborne Platform for Aerial
organism into their surroundings.
Surveillance-Beyond Horizon 201).
III. eDNA – based molecular methods is used
 The Aeronautical Development
to detect invasive species in aquatic
Establishment (ADE), Bengaluru, is
ecosystem.
responsible for its research and
development, and Hindustan Aeronautics

AMIGOS IAS 20
Which of the above given statement is/are  Neural networks are designed to work just
correct? like the human brain does. In the case of
1) I only 2) I and II only recognizing handwriting or facial
3) III only 4) II and III only recognition, the brain very quickly makes
Answer: 4 some decisions. For example, in the case of
Explanation: facial recognition, the brain might start
with “It is female or male?
 It is defined as the genetic material
60. The term ‘Geographic Information System’
obtained directly from environmental
recently seen in news, which of the following
samples (Soil, sediment, water, etc.)
statement is/are correct?
without any obvious signs of the biological
source material. It is an efficient, non- 1) It is a computer-based tool that examines
invasive and easy-to standardise sampling spatial relationships, patterns, and trends
approach. in geography.
 It originates from cellular material shed by 2) GIS mapping produces visualizations of
organisms (via skin, excrement, etc.) into geospatial information and help us
aquatic or terrestrial environments. understanding what is where.
3) e-Green Watch portal uses Geographic
 Coupled with sensitive, cost-efficient and
Information Systems (GIS) to analyse land
ever-advancing DNA sequencing
use in real time to track growth of
technology, the technique is increasingly
vegetation.
being used for biodiversity monitoring.
Select the correct answer using the code given
 Hence II and III are correct.
below:
59. “Neural Networks” is sometimes talked
1) Only one 2) Only two
about with reference to which one of the
following? 3) All three 4) None
1) It consists of networks made up of Answer: 3
neurons and synapses connected to and Explanation:
controlling tissues as well as impacting  Geographic Information Systems (GIS)
human thoughts and behaviour. store, analyze, and visualize data for
2) It is a means of communication that uses geographic positions on Earth’s surface.
signals encoded in light to transmit GIS is a computer-based tool that examines
information in various types of spatial relationships, patterns, and trends
telecommunications networks. in geography. It was used back in 1854
3) It is a method in artificial intelligence that (without computers of course!) to map a
teaches computers to process data in a disease outbreak in the City of London.
way that is inspired by the human brain. Fundamentally, we still use this type of
spatial analysis today but in a more
4) It refers to a place in the network where
sophisticated way. Hence Statement I is
incoming or outgoing traffic is silently
correct
discarded.
Answer: 3  GIS mapping produces visualizations of
geospatial information. The 4 main ideas of
Explanation:
Geographic Information Systems (GIS) are:
 A neural network is a method in artificial
 Create geographic data.
intelligence that teaches computers to
process data in a way that is inspired by the  Manage it in a database.
human brain. It is a type of machine  Analyze and find patterns.
learning process, called deep learning, that  Display it on a map.
uses interconnected nodes or neurons in a Hence Statement II is correct.
layered structure that resembles the As part of CAMPA, forest land is being diverted
human brain. to non-forest uses, so E-Green Watch is an
integrated and online Geographic Information

AMIGOS IAS 21
System (GIS) that will track the growth of  Oil spills that happen in rivers, bays and the
vegetation cover and encourage afforestation. ocean most often are caused by
All stakeholders and the general public will be accidents involving tankers, barges,
able to view it, and the data will be presented pipelines, refineries, drilling rigs, and
in real time. Hence Statement III is correct. storage facilities, but also occur from
61. The term “OneWeb” was in News recently recreational boats and natural disasters.
pertaining to India’s space mission. It is for:  Natural disasters like very heavy storms in
1) Enabling the connectivity between the the oceans, shaking of the sea floor due to
QUAD nations for Indo-Pacific region. earthquakes, and hurricanes have
2) Enabling connectivity between the G20 contributed to oil tanker ship accidents or
nations breakage/leakage of underground
3) Enabling connectivity for the corporate pipelines thereby causing colossal oil spills.
sectors.  Hence it can be caused by both
4) Enabling connectivity for governments, anthropological causes and natural
businesses, and communities causes. Hence statement II is incorrect.
Answer: 4  It causes hypothermia in birds and threats
Explanation: to the aquatic ecosystem.
One Web is a (Bharti group-backed) global  Crude oil ruins the insulating and
communications network, powered from waterproofing properties of feathers and
space, enabling connectivity for governments, fur of birds, and thus oil-coated birds
businesses, and communities and marine mammals may die from
62. Consider the following statements about the hypothermia.
oil spill  Hypothermia is a decrease in body
I. It refers to any uncontrolled release of temperature to below-normal levels.
crude oil and gasoline into the  Oil on ocean surfaces is harmful to many
environment. forms of aquatic life because it prevents
II. It can be caused only by an sufficient amounts of sunlight from
anthropological cause and not by natural penetrating the surface, and it also reduces
causes. the level of dissolved oxygen. Hence
III. It causes hypothermia in birds as crude oil statement III is correct.
ruins the insulating and waterproofing 63. Consider the following statements regarding
properties of feathers and fur. the Mangrove Alliance for Climate
Which of the above given statements is/are I. It is an intergovernmental alliance that
correct? works voluntarily as there are no real
1) I and II 2) II and III checks and balances to hold members
accountable.
3) I and III 4) I, II and III
II. India is a member of this alliance.
Answer: 3
Which of the above given statements is/are
Explanation:
correct?
 An oil spill refers to any uncontrolled
1) I only 2) II only
release of crude oil, gasoline, fuels, or other
oil by-products into the environment. 3) I and II 4) Neither I nor II
Answer:3
 Oil spills can pollute land, air, or
water, though it is mostly used for oceanic Explanation:
oil spills. Hence statement I is correct.  The Mangrove Alliance for Climate seeks
 It can be caused as a result of intensified to educate and spread awareness
petroleum exploration and production on worldwide on the role of mangroves in
continental shelves and the transport of curbing global warming and its potential as
large amounts of oils in vessels. a solution for climate change.

AMIGOS IAS 22
 It is an intergovernmental alliance that 65. Consider the following statements regarding
works voluntarily which means that there levels of biodiversity
are no real checks and balances to hold I. Genetic diversity ensures the survival of
members accountable. the population.
 Instead, the parties will decide their II. Species diversity refers to variety of living
commitments and deadlines regarding the and non-living things in a surrounding.
planting and restoring mangroves. III. Ecosystem diversity is influenced by the
 The members will also share expertise and nature of the ecosystem.
support each other in researching, Which of the above given statements is/are
managing, and protecting coastal correct?
areas. Hence statement I is correct. 1) I and II 2) II and III
 It was launched by the UAE and 3) I and III 4) I, II and III
Indonesia during the COP27 climate Answer: 3
summit in Sharm El Sheikh, Egypt. Explanation:
 It includes UAE, Indonesia, India, Sri Lanka,  Genetic diversity ensures the survival of
Australia, Japan, and Spain. the population as it ensures the survival of
 India is a member of this alliance. Hence individuals. It aims to ensure that some
statement II is correct. species survive drastic changes and thus
64. The term “Extended Reality” is sometimes carry-on desirable genes.
talked about with reference to which one of  It is concerned with the variation in genes
the following? within a particular species. It allows species
1) It is an umbrella term encapsulating to adapt to changing environments. Hence
Augmented, Virtual and Mixed Realities. statement I is correct.
2) It is a realistic simulation of an  Species diversity refers to the variety of
environment, including three-dimensional living organisms on earth. Species differ
graphics, by a computer system using from one another, especially in their
interactive software and hardware. genetic makeup, and do not interbreed in
3) It is a device aiming at reducing the time nature. It is the ratio of one species’
taken to achieve interstellar travel. population over the total number of
4) It is the phenomenon seen around black organisms across all species in the given
holes where reality and time undergoes biome. Zero represents infinite diversity
distortion. and one represents only one species
Answer: 1 present. Hence statement II is incorrect.
Explanation:  Ecosystem diversity refers to different
 Extended Reality (XR) is the combination of types of habitats.
human & computer-generated graphics  A habitat is the cumulative factor of the
interaction, which is in reality as well as the climate, vegetation, and geography of a
virtual environment. In basic terms, region. Ex: Grasslands and Corals.
Extended Reality is a superset of  Ecosystem diversity is influenced by the
Augmented Reality (AR), Virtual Reality nature of the ecosystem because the
(VR) & Mixed Reality (MR). climate changes are accompanied by the
 The concept of Extended Reality (XR) came change in vegetation as well and each
into the picture when technologies like species adapts itself to a particular kind of
Augmented and virtual reality, were being environment. Hence, the diversity of
used by developers and tech companies all species in the ecosystem is influenced by
across the globe. the nature of the ecosystem. Hence
statement III is correct.
]

AMIGOS IAS 23
2) Allows social media platforms to avoid
66. “Advanced Liquid Processing System” is a liability for posts made by users
technology most commonly used in the 3) Punishes the social media platforms for
which of the following situations? their liability for posts made by users
1) Sewage water recycling 4) Punishes the users for their liability for
2) Removal of Radioactive wastes from posts made on the social media platforms.
Water. Answer: 2
3) Use of Liquid Oxygen in Missile and Space Explanation:
Technologies. The government is reconsidering a key aspect
4) An alternative mechanism to refine crude of cyberspace – ‘safe harbor’, which is the
oil. principle that allows social media platforms to
Answer: 2 avoid liability for posts made by users. Safe
Explanation: harbor (originally from the US’s
 The Advanced Liquid Processing System Communications Decency Act) – as prescribed
(ALPS), a multi-nuclide removal system, under Section 79 of the IT Act, 2000 – is legal
removes various radioactive materials immunity that online intermediaries enjoy
from contaminated water. against content posted by users on their
platforms.
 When water comes into contact with
69. Consider the following statements regarding
melted fuel, fuel debris and other
e-waste regulations
radioactive substances, it becomes
contaminated. The contaminated water is I. India’s first e-waste clinic has been set up
then treated through a filtration process in Bengaluru, Karnataka.
known as Advanced Liquid Processing II. The Nairobi Declaration of the Basel
System (ALPS) to remove most of the Convention is related to the management
radioactivity before being stored. of electronic waste.
67. Honolulu Strategy is related to Which of the above given statements is/are
1) Mercury pollution correct?
2) Plastic pollution 1) I only 2) II only
3) Nitrogen pollution 3) Both I and II 4) Neither I nor II
4) Water pollution Answer:2
Answer: 2 Explanation:
Explanation:  India’s first e-waste clinic has been set up
in Bhopal, Madhya Pradesh. It is used for
 Honolulu Strategy is related to plastic
segregating, processing, and disposal of
pollution. National Oceanic and
waste from household and commercial
Atmospheric Administration (NOAA) in the
units.
United States and UNEP created the
Honolulu Strategy—a planning tool to  The Central Pollution Control Board will
reduce plastic pollution and its impacts. provide technical support at the unit and
the collected hazardous waste will then be
 The Honolulu Strategy is a framework for a
sent to Bengaluru for recycling.
comprehensive and global collaborative
effort to reduce the ecological, human  This would ensure the scientific handling
health, and economic impacts of marine and disposal of electronic waste generated
debris worldwide. from households and commercial
68. The government is reconsidering a key aspect establishments. Hence statement I is
of cyberspace – ‘safe harbor’. What does it incorrect.
mean?  The Nairobi Declaration of the Basel
1) Allow users to avoid liability for posts Convention is related to the management
made on social media platforms of electronic waste.

AMIGOS IAS 24
 It was adopted at COP9 of the Basel 1) I only 2) II and III only
Convention on the Control of the 3) I and III only 4) I, II and III
Transboundary Movement of Hazardous Answer: 3
Waste. It aimed at creating innovative Explanation:
solutions for the environmentally sound
 AK 203 Rifle is the most advanced version
management of electronic waste. Hence
of the AK-47 rifle. Hence statement I is
statement II is correct.
correct.
70. Consider the following statements:
 It is expected to replace the Indian Small
1) Speech recognition Arms System (INSAS) 5.56×45 mm assault
2) Medical diagnosis rifle, which is presently being used by
3) Predicting Traffic Patterns. Army, Navy and Air Force besides other
4) Predicting the possibility of Insurance security forces. INSAS rifles are not suitable
fraud. for use at high altitudes. Several other
Helping detect computer viruses issues with these rifles include gun
Which of the following functions can be jamming, oil leakage etc. Thus AK 203 will
performed through Machine Learning? replace them as they can be used in high
1) I, II and III 2) II, III, IV and V altitudes. Hence statement II is incorrect.
3) I, II and IV 4) I, II, III, IV and V  It is jointly produced by India and Russia.
Answer: 4 They had inked an agreement in 2021 for
the procurement of AK 203 assault rifles
Explanation:
through Korwa Ordnance Factory in Uttar
Machine learning can predict outcomes from Pradesh’s Amethi district. The deal has a
a business perspective, such as which of your clause for complete technology transfer
customers are likely to churn. It can also and the rifles will also be exported to
predict the likelihood of an insurance claim
friendly foreign nations. Hence statement
being fraudulent. The list of use cases for
III is correct.
machine learning that can be applied to is vast
72. Consider the following statements about
and may appear to be too complex to
Multiple Independent Re-entry Vehicle
comprehend quickly. However, the key
(MIRV)
element is identifying patterns with common
results. I. It is an exo-atmospheric ballistic missile
payload containing several warheads.
What are the Common Applications of
Machine Learning? II. Each warhead is capable of being aimed to
hit a different target
● Image Recogni on.
III. The Soviet Union was the first country to
● Speech Recogni on.
develop MIRV technology
● Predict Traffic Pa erns.
Which of the statements given above are
● E-commerce Product Recommendations.
correct?
● Self-Driving Cars.
1) I and II 2) II and III
● Catching Email Spam. 3) I and III 4) I, II and III
● Catching Malware.
Answer: 1
● Virtual Personal Assistant.
Explanation:
71. Consider the following statements about the
 A multiple independently targetable re-
Kalashnikov AK 203 Rifle:
entry vehicle (MIRV) is an exoatmospheric
I. It is the most advanced version of the AK- ballistic missile payload containing several
47 rifle. warheads, each capable of being aimed to
II. It cannot be used in high altitudes. hit a different target. Hence statement I
III. It is jointly produced by India and Russia. and II are correct.
Which of the above given statements is/are  The United States was the first country to
correct? develop MIRV technology, deploying a

AMIGOS IAS 25
MIRVed Intercontinental Ballistic Missile  BOD is limited to biodegradable materials.
(ICBM) in 1970 and a MIRVed Submarine-  Chemical Oxygen Demand (COD) measures
Launched Ballistic Missile (SLBM) in 1971. the amount of oxygen in parts per million
The Soviet Union quickly followed suit and required to oxidise organic (biodegradable
by the end of the 1970s had developed and non-biodegradable) and oxidizable
their own MIRV-enabled ICBM and SLBM inorganic compounds in the water
technology. Hence statement III is sample. Hence statement I is incorrect.
incorrect.  It is expressed in milligrams of oxygen per
73. Which of the following is/are applications of litre of water. Hence statement II is
drones? correct.
I. Mapping of landslide areas  Higher BOD indicates more oxygen is
II. Aerial photography of journalism required, which is less for oxygen-
III. Disaster management demanding species to feed on, and signifies
IV. Inspection of active volcanoes lower water quality. Inversely, low BOD
Choose the correct code: means less oxygen is being removed from
1) I, II and IV 2) II, III and IV the water, so water is generally
3) I, II and III 4) I, II, III and IV purer. Hence statement III is correct.
Answer: 4 75. Consider the following statements
Explanation: I. Lentic ecosystems include all ecosystems
with static or still water.
Applications of drones –
II. Lotic ecosystems include all fast-moving
Mapping of landslide areas
water bodies.
Aerial photography of journalism
III. Lentic ecosystems are also called the
Disaster management riverine ecosystem.
Border control surveillance IV. Lotic ecosystems are also called the
Forecasting storms and hurricanes lacustrine ecosystem.
Crop damage assessment Which of the statements given above are
Inspection of active volcanoes correct?
74. Consider the following statements about 1) I and IV 2) II and III
Biochemical Oxygen Demand (BOD) 3) II and IV 4) I and II
I. It is the amount of oxygen required to Answer: 4
oxidize organic and inorganic compounds Explanation:
in the water.
 Lentic ecosystems include all ecosystems
II. It is expressed in milligrams of oxygen per
with static or still water. Ex: Lakes and
litre of water.
Ponds. Hence statement I is correct.
III. Higher BOD indicates lower water quality.
 Lotic ecosystems include all fast-moving
Which of the above given statements is/are water bodies. Ex: Rivers and
correct? Streams. Hence statement II is correct.
1) I and II 2) II only  Lentic ecosystems are also called the
3) III only 4) II and III lacustrine ecosystem. Hence statement III
Answer: 4 is incorrect.
Explanation:  Lotic ecosystems are also called the
 Biochemical Oxygen Demand (BOD) is the riverine ecosystem. Hence statement IV is
amount of dissolved oxygen needed by incorrect.
bacteria in decomposing the organic 76. 'Totipotency' is best described as the ability
wastes present in water. of a
 Water pollution by organic wastes is 1) The capacity of individual cells to initiate
measured in terms of Biochemical Oxygen all lineages of the mature organism in a
Demand (BOD).

AMIGOS IAS 26
flexible manner directed by signals in the  It's designed to intercept missiles in the
embryo or cell culture environment atmosphere at high altitudes, between 50–
2) single cell to divide and produce all of the 80 km. Hence statement I is correct.
differentiated cells in an organism  S-400 Triumf air defense system is being
3) dying cell to regain life by slowing down purchased from Russia. Hence statement II
the process of oxidation. is correct.
4) new organism to develop from an  Akash mid-range surface-to-air missile
outgrowth or bud due to cell division at (SAM) system.
one particular site.  The missile system can target aircraft up to
Answer: 2 30 km away, at altitudes up to 18,000
Explanation: meters. Hence statement III is incorrect.
 Totipotency is the ability of a single cell to 78. It is India’s second-largest mangrove
divide and produce all of the differentiated ecosystem after the Sundarbans. This
cells in an organism. It was learned by National Park is a network of creeks and
scientists, during the 1950s, that whole canals which are inundated with waters from
plants could be regenerated from explants, rivers Dhamra and Patasala, forming a unique
i.e., any part of a plant taken out and grown ecosystem. It has one of the largest
in a test tube, under sterile conditions in populations of endangered saltwater
special nutrient media. This capacity to crocodiles in India.
generate a whole plant from any The above description is related to which of
cell/explant is called totipotency. the following National Park?
 Totipotency is set to become a key tool for 1) Galathea National Park
research and future medical applications. 2) Bhitarkanika National Park
Human development begins when a sperm 3) Gulf of Mannar Marine National Park
fertilizes an egg and the resulting fertilized 4) Rani Jhansi Marine National Park
egg creates a single totipotent cell, a
Answer: 2
zygote. The zygote divides numerous times
Explanation:
and forms cells that are the precursors to
the trillions of cells that will eventually  Bhitarkanika National Park:
constitute the human body.  Location: It is located in Kendrapara district
77. Consider the following statements with in the state of Odisha.
reference to the India’s air defence system:  It is India’s second-largest mangrove
I. Prithvi air defence system has the capacity ecosystem after the Sundarbans.
to engage targets at an altitude of 50 km.  It is a 145 Sq. km large national park which
II. S-400 Triumf air defense system is being was designated on 16th September 1998
purchased from Russia. and obtained the status of a Ramsar site in
III. Akash is a part of mid-range air to air August 2002, e., the second Ramsar site of
defense systems of India. the State after the Chilika Lake.
Which of the above statements is/are correct?  This National Park is a network of creeks
1) I and II only 2) I only and canals which are inundated with
waters from rivers Brahmani, Baitarani,
3) II and III only 4) I, II and III
Dhamra and Patasala, forming a unique
Answer: 1
ecosystem.
Explanation:
 Flora: Mangrove species, casuarinas, and
 The Prithvi Air Defence (PAD) system, also grasses like the indigo bush.
known as the Pradyumna Ballistic Missile
 Fauna: Bhitarkanika has one of the largest
Interceptor, is India's ballistic missile
populations of endangered saltwater
defense system.
crocodiles in India.
 The Gahirmatha Beach, which forms the
boundary of the park in the east, is the

AMIGOS IAS 27
largest colony of the Olive Ridley Sea a halt automatically when it notices
Turtles. another train on the same line
 Other mammals include monkeys, jackals, Which of the above given statements is/are
common langurs, otter, sambar deer, correct?
jungle cats, fox, Mongoose, wolfs, fishing 1) I and II 2) I and III
cats, hyenas, etc. 3) II and III 4) I, II and III
79. Consider the following statements with Answer: 4
reference to the Agni Missile system of India: Explanation:
I. It has been developed under the  Kavach System is an indigenously
Integrated Guided Missile Development developed Automatic Train Protection
Programme of DRDO. (ATP) system. Kavach was developed by the
II. Agni II is an intermediate range surface-to- Research Design and Standards
surface nuclear-capable missile. Organisation (RDSO) under Indian Railway
III. Agni V is a canister launch solid fuelled (IR) in collaboration with Medha Servo
Inter-Continental Ballistic Missile (ICBM). Drives Pvt Ltd, HBL Power Systems Ltd and
Which of the above statements is/are correct? Kernex Microsystems. Hence statement I is
1) I and III only 2) II only correct.
3) I and II only 4) I, II and III  It is a set of electronic devices and Radio
Answer: 4 Frequency Identification devices installed
Explanation: in locomotives, in the signalling system as
well the tracks, that talk to each other using
 It has been developed under the Integrated
ultrahigh radio frequencies to control the
Guided Missile Development Programme
brakes of trains and also alert drivers, all
of DRDO. Hence statement I is correct.
based on the logic programmed into them.
 Agni II is an intermediate range surface-to- Hence statement II is correct.
surface nuclear-capable missile. It is a two-
 The system can alert the loco pilot, take
stage, solid-fuelled, medium-range ballistic
control of the brakes and bring the train to
missile (MRBM) with a range of 2,000–
a halt automatically when it notices
3,500 kilometres. It has a payload capacity
another train on the same line within a
of 1,000 kilograms and a flight ceiling of
prescribed distance. The device also
405 kilometres. Hence statement II is
continuously relays the signals ahead to
correct.
the locomotive, making it useful for loco
 Agni-V (Fire) is a land based nuclear pilots in low visibility. Hence statement III
capable intercontinental ballistic is correct.
missile that has a range of more than 7,000
81. Consider the following statements regarding
kilometres. Hence statement III is correct.
Light Combat Aircraft Tejas Mk2
80. Consider the following statements with
I. It’s mission endurance for fighting a war is
respect to ‘Kavach System’
300 minutes
I. It is an indigenously developed Automatic
II. It is specifically designed to carry only air-
Train Protection (ATP) system developed
to-air weapons.
by the Research Design and Standards
Organisation Which of the above given statements is/are
correct?
II. It is a set of electronic devices and Radio
Frequency Identification devices installed 1) I only 2) II only
in locomotives as well the tracks which 3) Both I and II 4) Neither I nor II
communicate through ultra-high radio Answer: 4
frequencies Explanation:
III. The system can alert the loco pilot, take  The Tejas Mk2 is a fighter aircraft
control of the brakes and bring the train to developed in India that can carry eight
Beyond-Visual-Range (BVR)missiles

AMIGOS IAS 28
simultaneously. It is an upgraded version of Research and Development Organisation
the LCA Tejas Mk1, which has improved (DRDO) and is manufactured by Bharat
range and mission endurance. The mission Dynamics Ltd (BDL). Hence statement III is
endurance for fighting a war was 57 incorrect.
minutes for LCA Tejas Mk1, but it is 120 83. With reference to ‘Air Independent
minutes for LCA Tejas Mk2. Hence Propulsion (AIP) System’ used in submarines,
statement I is incorrect. consider the following statements
 The Light Combat Aircraft is designed I. Statement 1: It allows the submarines to
to carry only air-to-air, air-to-surface, stay in water by eliminating the need to
precision-guided, weapons. It has the air- come to the surface of the water to charge
to-air refuelling capability. Hence their batteries
statement II is incorrect. II. Statement 2: AIP systems generate
82. Consider the following statements with electricity, which in turn drives an electric
respect to ‘Varunastra’ motor for propulsion or recharges the
I. It is ship launched thermally-propelled boat’s batteries
anti-submarine torpedo capable of Which one of the following is correct with
targeting quiet submarines respect to the above statements?
II. The torpedo’s guidance system uses 1) Both Statement-I and Statement-II are
active-passive acoustic homing and correct and Statement II is the correct
GPS/NavIC satellite guidance explanation for Statement-I
III. It was designed and manufactured by 2) Both Statement-I and Statement-II are
Naval Science and Technological correct and Statement II is not the correct
Laboratory (NSTL) under the Defence explanation for Statement-I
Research and Development Organisation 3) Statement-I is correct but statement II is
(DRDO) incorrect
Which of the above given statements is/are 4) Statement -I is incorrect but Statement -II
correct? is correct
1) I and II 2) II only Answer:4
3) I and III 4) I, II and III Explanation:
Answer: 2  AIP system allows the submarines to stay
Explanation: for longer hours in water. The submarines
 Varunastra is a ship launched, heavy need to come to the surface of the water to
weight, electrically propelled anti- charge their batteries. This is reduced by
submarine torpedo capable of targeting AIP System. It decreases the noise levels
quiet submarines, both in deep and made by the submarines. This makes it
shallow waters in an intense counter- hard to detect the submarines. Hence
measures environment. Hence statement I statement I is incorrect.
is incorrect.  AIP is mostly implemented as an auxiliary
 The torpedo’s guidance system uses active- source, with the traditional diesel engine
passive acoustic homing, wire guidance, handling surface propulsion. Most of these
and GPS/NavIC satellite guidance. systems generate electricity, which in turn
Weighing around 1.5 tonnes, it can carry a drives an electric motor for propulsion or
250 kg (550 lb) high-explosive warhead and recharges the boat’s batteries. AIP allows
has a maximum operating depth of 600 longer submergence than a conventionally
metres (2,000 ft). It has long range with propelled submarine. Hence statement II is
multi manoeuvring capabilities. Hence correct.
statement II is correct. 84. ‘Raccoon Stealer’ seen in news recently is
 It was designed and developed by Vizag- related to
based Naval Science and Technological 1) Phishing software 2) Spoofing
Laboratory (NSTL) under the Defence 3) DNS tunnelling 4) Malware

AMIGOS IAS 29
Answer: 4  Hence statement I is incorrect.
Explanation:  States can declare areas adjacent to
Raccoon Stealer is a kind of malware that National parks and Sanctuaries
steals various data from an infected computer. as Conservation Reserve, for protecting
It is a classic example of information-stealing flora and fauna, and their habitat. Hence
malware, which cybercriminals typically use to statement II is correct.
gain possession of sensitive data saved in  It entrusts the Chief Wildlife Warden to
users’ browsers and cryptocurrency wallets. control and maintain all sanctuaries in a
Like most info-stealers, Raccoon Stealer is state. The Chief Wildlife Warden
purchasable. is appointed by the state. Hence
85. Consider the following statements regarding statement III is correct.
the Wildlife (Protection) Amendment Act of  For general violations (increases to Rs
2022 1,00,000 from Rs25,000) and specially
I. The number of schedules has been protected animals (increases to 25,000
increased from four to six. from Rs 10,000).
II. States can declare areas adjacent to  People possessing live specimens of
national parks as a conservation reserves. scheduled animals must obtain a
III. It entrusts the Chief Wildlife Warden to registration certificate from the
control and maintain all sanctuaries in a Management Authority.
state.  It provides to voluntarily surrender any
IV. It provides to voluntarily surrender any captive animals, without any
captive animals, without any compensation. Hence statement IV is correct.
compensation. 86. Consider the following statements about
Which of the above given statements are Supercapacitors
correct? I. Supercapacitors are electronic devices
1) I, III and IV 2) II, III and IV which are used to store extremely large
3) I, II and III 4) I, II, III and IV amounts of electrical charge.
Answer: 2 II. Supercapacitor has high power density and
Explanation: long durability compared to Lithium-Ion
 The Wildlife (Protection) Amendment Act batteries.
of 2022 seeks to conserve and protect Which of the above given statements is/are
wildlife through better management of correct?
protected areas 1) I only 2) I and II
and rationalise schedules that list out 3) II only 4) Neither I nor II
species under the Wildlife (Protection) Act, Answer: 2
of 1972. Explanation:
 The number of schedules has been  Capacitors, while storing electric charge for
decreased from six in the Wildlife much longer, cannot discharge energy
(Protection) Act, of 1972 to four. constantly.
Animal species that will enjoy  Supercapacitors overcomes these
Schedule I
the highest level of protection shortfalls by combining the properties of
Animal species that will be capacitors and batteries into one device.
Schedule II subject to a lesser degree of  Supercapacitors - Supercapacitors are
protection electronic devices which are used to store
extremely large amounts of electrical
Schedule III Protected Plant species
charge.
Specimens listed in the  It has significant advantages such as high-
Schedule IV Appendices under CITES power density, long durability, and
(scheduled specimens) ultrafast charging characteristics as

AMIGOS IAS 30
compared to conventional capacitors and particle collider. Hence, statement I is
Lithium-Ion batteries (LIB). correct.
Hence both the statements are correct.  In its operational state, it fires two beams
87. Consider the following statements of protons almost at the speed of light in
I. The Kuiper Belt is a region of the solar opposite directions inside a ring of
system that exists beyond the inner superconducting electromagnets. The
planets, extending from the orbit of magnetic field created by the
Jupiter. superconducting electromagnets keeps the
II. The Asteroid Belt has the maximum protons in a tight beam and guides them
number of known asteroids and is found along the way as they travel through beam
between Mars and Jupiter in the solar pipes and finally collide.
system.  Just before the collision, another type of
Which of the above given statements is/are magnet is used to ‘squeeze’ the particles
correct? closer together to increase the chances of
1) I only 2) II only collisions. The particles are so tiny that the
task of making them collide is akin to firing
3) Both I and II 4) Neither I nor II
two needles 10 km apart with such
Answer: 2
precision that they meet halfway. Hence,
Explanation: statement II is incorrect.
 The Kuiper Belt is a region of the solar  ATLAS is the largest general-purpose
system that exists beyond the eight major particle detector experiment at the
planets, extending from the orbit of LHC. Hence, statement III is correct.
Neptune. Hence, statement I is incorrect.
 In the LHC’s first run, the Higgs boson was
 The Asteroid Belt has the maximum discovered. The Higgs boson is the
number of known asteroids and is fundamental particle associated with
found between Mars and Jupiter in the the Higgs field, a field that gives mass
solar system. Hence, statement II is to other fundamental particles such
correct. as electrons and quarks. Hence, statement
88. Consider the following statements about the IV is correct.
‘Large Hadron Collider (LHC)’ 89. Consider the following statements regarding
I. It is a giant, complex machine built to study the NASA’s Space Missions
particles that are the smallest known 1. Artemis I is a crewed space mission to the
building blocks of all things. satellite moon.
II. In its operational state, it fires two beams 2. Perseverance Rover is a space mission to
of electrons almost at the speed of light in planet Mars.
opposite directions inside a ring of
3. Messenger is a space mission to planet
superconducting electromagnets.
Mercury.
III. ATLAS is the largest general-purpose
4. Mariner 2 is a space mission to planet
particle detector experiment at the LHC.
Venus.
IV. In the LHC’s first run, the Higgs boson was
Which of the above statements is/are correct?
discovered.
1) Only One 2) Only Two
Choose the correct code:
3) Only Three 4) All Four
1) I and II 2) III and IV
Answer: 3
3) II, III and IV 4) I, III and IV
Explanation:
Answer: 4
 The National Aeronautics and Space
Explanation:
Administration (NASA) is an independent
 The Large Hadron Collider is a giant, agency of the executive branch of the
complex machine built to study particles United States federal government
that are the smallest known building blocks responsible for the civilian space program,
of all things. It is the world’s most powerful as well as aeronautics and aerospace
AMIGOS IAS 31
research. It was established under the  While dark matter attracts and holds
National Aeronautics and Space Act of galaxies together, dark energy repels and
1958 and is headquartered in Washington, causes the expansion of the
DC, USA universe. Hence, statement III is correct.
 Artemis I is an uncrewed space mission to 91. Consider the following statements
the moon. Hence, statement I is incorrect. I. Space debris refers to man-made objects
 Perseverance Rover is a space mission to in Earth’s orbit that no longer serve a
Mars. Hence, statement II is correct. useful purpose.
 Messenger is a space mission II. Project NETRA is an early warning system
to Mercury. Hence, statement III is in space launched by the Indian Space
correct. Research Organisation to detect debris
 Mariner 2 is a space mission and other hazards to Indian satellites.
to Venus. Hence, statement IV is correct. III. Kessler Syndrome refers to a
90. Consider the following statements phenomenon where humans on earth
I. Unlike normal matter, dark matter does might get physically hurt by increasing
not absorb, reflect or emit light, making it density of space debris
extremely hard to spot. IV. European Space Agency has launched
II. The dark matter is more in abundance as the Clean Space initiative, aimed at
compared to the dark energy in the reducing the amount of space debris.
universe Which of the above given statements is/are
III. While dark matter attracts and holds correct?
galaxies together, dark energy repels and 1) I and II 2) II, III and IV
causes the expansion of the universe. 3) I, II and IV 4) I, II and III
Which of the above given statements is/are Answer: 3
correct? Explanation:
1) I and II 2) I and III  Space debris refers to man-made objects in
3) II and III 4) I, II and III Earth’s orbit that no longer serve a useful
Answer: 2 purpose. This includes defunct satellites,
Explanation: spent rocket stages, and fragments of
debris from collisions or other events. Of
 Dark matter is made up of particles that do
known and tracked space junk 70 percent
not interact through electromagnetic
is in low-Earth orbit, which extends about
interactions i.e. they do not have any
1,250 miles (2,000 km) above the Earth’s
charge. So, these are dark particles,
surface. Hence, statement I is correct.
namely because they do not emit light,
which is an electromagnetic phenomenon,  Project NETRA is an early warning system
and matter because they possess mass like in space launched by the Indian Space
normal matter and hence interact through Research Organisation (ISRO) to detect
gravity. Dark energy is an unknown form of debris and other hazards to Indian
energy that affects the universe on the satellites.
largest scales. The first observational  ISRO has also set up a Centre for Space
evidence for its existence came Debris Research to monitor and mitigate
from measurements of supernovae, which the threat of space debris and the System
showed that the universe does not expand for Safe and Sustainable Operations
at a constant rate and rather, the Management (IS 4 OM) to continually
expansion of the universe is monitor objects posing collision threats,
accelerating. Hence, statement I is correct. predict the evolution of space debris, and
 The universe is made of about 27% dark mitigate the risk posed by space
matter and 68% dark energy. Hence, debris. Hence, statement II is correct.
statement II is incorrect.  The Kessler syndrome (also called the
Kessler effect,[1][2] collisional cascading,

AMIGOS IAS 32
or ablation cascade), proposed by NASA hundreds of degrees hotter than Earth.
scientist Donald J. Kessler in 1978, is a According to NASA, more than 5,000
scenario in which the density of objects in exoplanets have been discovered. Hence,
low Earth orbit (LEO) due to space pollution statement II is incorrect.
is high enough that collisions between  Exoplanets cannot be seen directly through
objects could cause a cascade in which telescopes from the Earth because they are
each collision generates space debris that hidden by the bright glare of the stars they
increases the likelihood of further orbit. So, scientists rely on indirect
collisions. Hence, statement III is methods, such as the transit method,
incorrect. which is measuring the dimming of a star
 The European Space Agency has launched that happens to have a planet pass in front
the Clean Space initiative, aimed at of it, and gravitational microlensing in
reducing the amount of space debris. which light from a distant star is bent
Hence, statement IV is correct. and focused by gravity as a planet passes
92. Consider the following statements between the star and Earth. Hence,
I. Exoplanets are the planets that orbit stars statement III is correct.
other than sun and are beyond our solar  The study of exoplanets helps in
system. determining the habitable zone of the
II. LHS 475 b is a new terrestrial rocky discovered space and helps us piece
exoplanet discovered recently by the together information about our planetary
Indian Space Research Organisation. system and origin. Hence, statement IV is
III. Exoplanets cannot be seen directly through correct.
telescopes from the Earth due to their 93. Consider the following statements about
distance the Indian Data Relay Satellite System
IV. The study of exoplanets helps in (IDRSS)
determining the habitable zone of the I. It helps to track and be constantly in touch
discovered space. with Indian satellites, in particular to those
Which of the above given statements is/are in the low earth orbits.
correct? II. Currently, only the US and Russia have
1) I, III and IV 2) II, III and IV their own data relay satellite systems.
3) I, II and III 4) I, II, III and IV Which of the above given statement is/are
Answer:1 incorrect?
Explanation: 1) I only 2) II only
3) Both I and II 4) Neither I nor II
 Exoplanets are planets that orbit other
stars and are beyond our solar Answer: 2
system. They come in a host of different Explanation:
sizes. They can be gas giants bigger than  The Indian Data Relay Satellite System
Jupiter or as small and rocky as Earth. They (IDRSS) helps to track and be constantly in
are also known to have different kinds of touch with Indian satellites, in particular
temperatures from boiling hot to freezing those in low-earth orbits. It is an Indian
cold. Hence, statement I is correct. geostationary data relay satellite system
 LHS 475 b is a new terrestrial rocky consisting of two geostationary satellites.
exoplanet discovered by the National IDRSS will facilitate the relay of information
Aeronautics and Space Administration between various Indian spacecraft, in-flight
(NASA). It is roughly the same size as launch vehicle monitoring, and assist
Earth, its diameter is 99% the same as Indian Human Spaceflight
Earth. LHS 475 b is about 41 light-years Programme. Hence, statement I is correct.
away from Earth in the constellation  Currently, the US, Russia, China, and
Octans and differs from the Earth in that it Europe have their data relay
completes an orbit in just two days and is satellites. Hence, statement II is incorrect.

AMIGOS IAS 33
94. Consider the following statements about the Explanation:
Hyperspectral Imaging Satellite (HysIS)  IVF is a complex process that involves
I. It was launched by the Indian Space retrieving eggs from ovaries and manually
Research Organisation (ISRO) to study the combining them with sperm in a lab for
atmosphere of Earth. fertilization. Hence statement I is
II. Its primary goal is to study the Earth’s incorrect.
surface in visible, near-infrared, and  Artificial insemination technique
shortwave infrared regions of the introduces previously selected semen into
electromagnetic spectrum. the woman’s uterus that has been
Which of the above given statement is/are prepared by stimulating ovulation. Hence
correct? statement II is incorrect.
1) I only 2) II only 96. Consider the following statements regarding
3) Both I and II 4) Neither I nor II ‘SANDES’:
Answer: 3 I. It is an instant messaging platform on the
Explanation: lines of WhatsApp.
 The Hyperspectral Imaging Satellite II. It allows a user to mark a message as
(HysIS) was launched by the Indian Space confidential, this safety feature makes the
Research Organisation. Hence, statement I message unshakeable with others.
is correct. III. It is developed by the National Informatics
 Its primary goal is to study the Earth’s Centre under the aegis of the Ministry of
surface in visible, near-infrared, and Science and Technology.
shortwave infrared regions of the Which of the given above is/are correct?
electromagnetic spectrum. It can see in 55 1) I only 2) III only
spectral or colour bands from 630 km 3) I and III only 4) I, II and III
above the ground. Hyperspectral imaging Answer: 1
allows the distinct identification of objects, Explanation:
materials, or processes on Earth by reading  SANDES is an instant messaging platform
the spectrum for each pixel of a scene from on the lines of WhatsApp. Hence
space. It can be highly useful in marking out statement I is correct.
a suspect object or person and separating
 It allows a user to mark a message as
it from the background. This could aid in
confidential, which will allow the recipient
detecting transborder or other stealthy
to be made aware that the message should
movements. Hence, statement II is
not be shared with others. The confidential
correct.
tag, however, does not change the way the
95. Consider the following statements regarding
message is sent from one user to another.
Artificial insemination and In Vitro
Hence statement II is incorrect.
Fertilization:
 It is developed by the National Informatics
I. In Vitro Fertilization technique introduces
Centre under the aegis of the Ministry of
previously selected semen into the
Electronics and Information Technology.
woman’s uterus that has been prepared
Hence statement III is correct.
by stimulating ovulation.
97. Consider the following statements with
II. Artificial insemination technique consists
reference to ‘Internet of Things (IoT)’:
of retrieving a woman’s eggs to be
fertilised in the laboratory.  IoT generally refers to scenarios where
network connectivity and computing
Which of the statements given above is/are
capability extends to objects, sensors and
correct?
everyday items.
1) I only 2) II only
 IoT devices and services can serve potential
3) Both I and II 4) Neither I nor II
entry points for cyber-attack and expose
Answer: 4 user-data to theft.

AMIGOS IAS 34
 IoT amplifies concerns about the potential  It may result in the formation of super
for increased surveillance and tracking. bugs.
Which of the statements given above is/are  A superbug is usually defined as a
correct? microorganism that’s resistant to
1) I and II only 2) II and III only commonly used antibiotics. Hence
3) III only 4) I, II and III statement II is correct.
Answer: 4 99. Which of the following statements correctly
Explanation: explains the mechanism of Pascal's law in
physics?
 The Internet of Things (IoT) is a system of
physical objects that are connected to the 1) Pressure applied to an enclosed fluid is
internet and can exchange data with other distributed equally in all directions.
devices and systems. These objects can 2) The density of a fluid increases with depth.
range from household items to industrial 3) The flow rate of a fluid is inversely
tools. proportional to its viscosity.
 IoT is the interlinking of digital devices, 4) Buoyant force on an object is equal to the
people, machines, appliances, and other weight of the displaced fluid.
objects with one another through wireless Answer: 1
networks. Explanation:
 It allows machines and people to be  Pascal's Law, also known as Pascal's
connected to each other and communicate Principle, is a fundamental principle in fluid
as well. mechanics. It states that when pressure is
 It is considered the future of the internet. applied to an enclosed fluid in a confined
This version of the internet is about data space, the resulting increase in pressure is
that is created by things. transmitted equally in all directions within
 IoT devices and services can serve potential the fluid. This means that if you push or
entry points for cyber-attack and expose apply pressure to one part of the fluid, that
user-data to theft. IoT amplifies concerns pressure is not localized; instead, it
about the potential for increased propagates uniformly throughout the
surveillance and tracking. entire fluid volume. This principle is the
Hence all three statements are correct. reason hydraulic systems work efficiently.
98. Consider the following statements with  For example, in a hydraulic car brake
reference to the Anti-microbial resistance: system, a small force applied to the brake
pedal creates pressure in the brake fluid,
I. It is a gain of susceptibility of bacteria to
which is then transmitted to all parts of the
the killing properties of an antibiotic
brake system. As a result, the brake
agent.
callipers can apply a strong force to the
II. It may result in the formation of super
brake pads, allowing the vehicle to stop
bugs.
effectively. Pascal's Law is essential in
Which of the following statement is/are various applications, including hydraulic
correct? machinery, lift systems, and many other
1) I only 2) II only engineering and industrial processes that
3) Both I and II 4) Neither I nor II rely on the transmission of force through
Answer: 2 fluids.
Explanation: 100. Consider the following statements
 Antibiotic resistance is a loss of Statement I: DNA is responsible for controlling
susceptibility of bacteria to the killing or the hereditary characteristics of living cells.
growth-inhibiting properties of an Statement II: It is because it performs the
antibiotic agent. Hence statement I is function of storing and transferring genetic
incorrect. information.

AMIGOS IAS 35
Which one of the following is correct in the nuclei of the sperm and egg, in the early
respect of the above statements? stages of embryo development.
1) Both Statement I and Statement II are 'Pronuclear Transfer' is a human
correct and Statement II is the correct reproductive technology, which is used for
explanation for Statement I creating a 'three-parent baby' for the
2) Both Statement I and Statement II are prevention of mitochondrial diseases in
correct and Statement II is not the correct offspring. The technique involves
explanation for Statement I intervening in the fertilization process to
3) Statement I is correct but Statement II is remove faulty mitochondria which can
incorrect · cause fatal heart problems, liver failure,
brain disorders, blindness, muscular
4) Statement I is incorrect but Statement II is
dystrophy etc.
correct
Answer: 1  The treatment is known as 'three-parent'
IVF because the babies born from
Explanation:
genetically modified embryos would have
 DNA is responsible for storing and DNA from a mother, a father and a woman
transferring genetic information while RNA donor.
directly codes for amino acids and acts as a
102. Which country has launched a 'sniffing robot'
messenger between DNA and ribosomes to
for early detection of diseases and improving
make proteins. However, studies showed
security checks?
that RNA replaces DNA in some viruses like
1) India 2) Australia
Tobacco Mosaic virus, QB bacteriophage,
etc. So, not every organism has DNA as its 3) Russia 4) Israel
hereditary material, some have RNA. But Answer: 4
DNA is the predominant one and in the Explanation:
majority of higher-level organisms, RNA  Scientists from Israel have developed a
plays the role of a messenger. sniffing robot which is equipped with a
 DNA, or deoxyribonucleic acid, is a biological sensor.
molecule that plays a fundamental role in  The robot uses the antennae of a locust to
controlling the hereditary characteristics of diagnose disease and improve security
living cells. This hereditary control is checks.
achieved through a complex interplay of  Locust’s smell with their antennae. The
genetic information encoded in the DNA scientists will use the locust’s antennae to
molecule. hone the computer’s scent detection
101. Which of the following technologies is used abilities.
for creating a 'three-parent baby' for the  This could help advance disease diagnosis
prevention of mitochondrial diseases in and improve security checks.
offspring? 103. Consider the following statements about
1) RNA interference Multimodal Artificial Intelligence (AI)
2) IVF technique I. It refers to the integration of multiple
3) Recombinant DNA technology modes of information or sensory data to
4) Pronuclear Transfer facilitate human-like reasoning and
Answer: 4 decision-making.
Explanation: II. Gemini is a multimodal large language
 Pronuclear transfer, also known as model being developed by the OpenAI.
mitochondrial replacement therapy (MRT), Which of the above given statements is/are
is a specialized reproductive technique correct?
used to prevent the transmission of certain 1) I only 2) II only
mitochondrial diseases from mother to 3) Both I and II 4) Neither I nor II
child. This procedure involves the Answer: 1
manipulation of the pronuclei, which are
AMIGOS IAS 36
Explanation:  IGMS2.0 Dashboard has been
 The multimodal model artificial intelligence implemented by IIT Kanpur. CPGRAMS is an
refers to the integration of multiple modes online platform available to the citizens
of information or sensory data to facilitate 24x7 to lodge their grievances to the public
human-like reasoning and decision- authorities on any subject related to
making. service delivery. It is a single portal
 It revolutionizes the way AI systems connected to all the
process and interpret information by Ministries/Departments of Government of
seamlessly integrating various sensory India and States. Every Ministry and States
modalities. have role-based access to this system.
 Multimodal systems allow users to engage  CPGRAMS is also accessible to the citizens
with AI in several ways. Unlike through standalone mobile application
conventional Al models, which focus on a downloadable through Google Play store
single data type, multimodal Al systems and mobile application integrated with
have the capability to simultaneously UMANG.
comprehend and utilize data from diverse Hence both the statements are correct.
sources such as text, images, audio, and 105. Consider the following statements about
video. Hence statement I is correct. Continuously Operating Reference Stations
 Gemini is multimodal large language model (CORS) Network
being developed by the google. Gobi is I. It provides precise Location based services,
multimodal AI being developed by the which are capable of providing centimetre
OpenAI. Hence statement II is incorrect. level positioning services in real-time.
104. Consider the following statements about II. These stations are operated by the Survey
Intelligent Grievance Monitoring System of India, the National Survey and Mapping
(IGMS) 2.0 Organization of the country, under
I. The IGMS 2.0 is a major upgradation of Ministry of Science & Technology.
Centralised Public Grievance Redress and Which of the above given statements is/are
Monitoring System portal with artificial correct?
intelligence and machine learning 1) I only 2) II only
capabilities. 3) Both I and II 4) Neither I nor II
II. Intelligent Grievance Monitoring System Answer: 3
(IGMS) 2.0 is implemented by IIT Kanpur. Explanation:
Which of the above statements is/are  It is capable of precise Location based
incorrect? services, which are capable of providing
1) I only 2) II only centimetre level positioning services in
3) Both I and II 4) Neither I nor II real-time These stations are capable of
Answer: 4 providing Real Time Positioning Service
Explanation: through Real Time-Kinematic with an
 The IGMS 2.0 is a major upgradation of accuracy of + / - 3cm. The CORS network is
Centralised Public Grievance Redress and available 24 /7 throughout the year.
Monitoring System (CPGRAMS) portal with Significance CORS based precision services
artificial intelligence (AI) and machine will boost auto navigation and machine
learning (ML) capabilities. control-based solution in Agriculture,
Mining, Construction, Transport and Civil
 The Dashboard provides instant tabular
Aviation sector.
analysis of Grievances Filed & Disposed,
State-wise & District-wise Grievances Filed  This will open new domains of innovation
& Ministry-wise data. The Dashboard will and research and create a Geospatial based
also help the officials identify the root ecosystem in these sectors. CORS data will
cause of the grievance. also aid in various scientific studies like
Upper Atmosphere and Space weather

AMIGOS IAS 37
studies, Meteorology and weather  It consists of both L-band and S-band
forecast, Plate motion and Tectonic synthetic aperture radar (SAR)
studies, Seismology and Hydrology etc. instruments, which makes it a dual-
 These stations will be operated by the frequency imaging radar satellite. NISAR
Survey of India. The Survey of India has set will be the first satellite mission to use two
up more than 1,000 CORS stations across different radar frequencies (L-band and S-
India. Survey of India It is the National band) to measure changes in our planet's
Survey and Mapping Organization of the surface. SAR is capable of penetrating
country under the Department of Science clouds and can collect data day and night
& Technology, Ministry of Science & regardless of the weather conditions.
Technology.  It also consists of a large 39-foot stationary
 It is the oldest scientific department of antenna reflector made of a gold-plated
India. It was set up in 1767 and has evolved wire mesh which will be used to focus “the
rich traditions over the years It is assigned radar signals emitted and received by the
the role as the nation's Principal Mapping upward-facing feed on the instrument
Agency, Survey of India bears a special structure.
responsibility to ensure that the country's Hence both the statements are correct.
domain is explored and mapped suitably. 107. With reference to ‘Navigation with Indian
Hence both the statements are correct. Constellation (NavIC)’, consider the following
106. Consider the following statements about statements
NISAR Statement 1: It is global navigation satellite
I. NISAR is a Low Earth Orbit (LEO) system developed by ISRO.
observatory jointly developed by NASA Statement 2: It comprises 7 satellites, three in
and ISRO. geostationary and four in inclined
II. NISAR is the first satellite mission to use geosynchronous orbits.
two different radar frequencies to Which one of the following is correct in
measure changes in our planet's surface. respect of the above statements?
Which of the above given statements is/are 1) Both Statement-I and Statement-II are
correct? correct and Statement II is the correct
1) I only 2) II only explanation for Statement-I
3) Both I and II 4) Neither I nor II 2) Both Statement-I and Statement-II are
Answer:3 correct and Statement II is not the correct
Explanation: explanation for Statement-I
 NISAR is a Low Earth Orbit (LEO) 3) Statement-I is correct but statement II is
observatory jointly developed by NASA and incorrect
ISRO. It is an SUV-size satellite weighing 4) Statement -I is incorrect but Statement -II
2,800 kilograms. NASA has provided the L- is correct
band radar, GPS, a high-capacity solid-state Answer: 4
recorder to store data, and a payload data Explanation:
subsystem. ISRO has provided the S-band  Navigation with Indian Constellation
radar, the GSLV launch system, and (NavIC) To meet the positioning, navigation
spacecraft. and timing requirements of the nation,
 It will measure Earth’s changing ISRO has established a regional navigation
ecosystems, dynamic surfaces, and ice satellite system called Navigation with
masses, providing information about Indian Constellation (NavIC). NavIC was
biomass, natural hazards, sea level rise, erstwhile known as Indian Regional
and groundwater. NISAR will observe Navigation Satellite System (IRNSS).
Earth’s land and ice-covered surfaces  NavIC is designed with a constellation of 7
globally with 12-day regularity on satellites and a network of ground stations
ascending and descending passes. operating 24 x 7. Three satellites of the

AMIGOS IAS 38
constellation are placed in geostationary II. The fire and forget missile have a
orbit, and four satellites are placed in minimum range of 500 metres and a
inclined geosynchronous orbit. maximum range of 7 kilometres.
 NavIC offers two services: Standard Which of the above given statements is/are
Position Service (SPS) for civilian users and correct?
Restricted Service (RS) for strategic users. 1) I only 2) II only
NavIC coverage area includes India and a 3) Both I and II 4) Neither I nor II
region up to 1500 km beyond Indian Answer:2
boundary. Explanation:
 NavIC signals are designed to provide user  It is a helicopter-launched anti-tank guided
position accuracy better than 20m and missile (ATGM) system. It is a short-range
timing accuracy better than 50ns. NavIC air-to-surface missile. It was indigenously
SPS signals are interoperable with the
developed by the Defence Research and
other global navigation satellite system Development Organisation (DRDO). Hence
(GNSS) signals namely GPS, Glonass, statement I is incorrect.
Galileo and BeiDou.
 It is equipped with Imaging Infra-Red (IIR)
108. Consider the following statements with
seeker and operates in “lock on before
reference to ‘SARAS 3’
launch” mode. The fire and forget missile
I. It is an x-ray telescope to observe have a minimum range of 500 metres and
luminous galaxies. a maximum range of 7 kilometres.
II. It is developed by Indian Space Research  It can be launched from an altitude of up to
Organization. 4 kilometres and can hit targets moving at
Which of the above given statements is/are speeds of up to 70 kilometres per hour. The
incorrect? missile can engage targets both in direct hit
1) I only 2) II only mode as well as top attack mode. It works
3) Both I and II 4) Neither I nor II in all kinds of weather, during the day or
Answer:3 night, and in various types of terrain, such
Explanation: as deserts, plains, hills and forests. Hence
 SARAS aims to design, build and deploy in statement II is correct.
India a precision radio telescope to detect 110. ‘Transiting Exoplanet Survey Satellite (TESS)’
extremely faint radio wave signals from the recently seen in news is a mission of
depths of time, from our “Cosmic Dawn” 1) European Space Agency
when the first stars and galaxies formed in 2) National Aeronautics and Space
the early Universe. Hence statement I is Administration (NASA)
incorrect. 3) Canadian Space Agency
 SARAS is a niche high-risk high-gain 4) Indian Space Research Organisation
experimental effort of Raman Research Answer: 2
Institute (RRI) The RRI is an autonomous Explanation:
research institute engaged in research in Transiting Exoplanet Survey Satellite (TESS) is
basic sciences. The institute was founded in a NASA mission to discover exoplanets around
1948 by the Indian physicist and Nobel nearby bright stars. TESS circles Earth in a
Laureate Sir C V Raman. Hence statement unique high Earth orbit of 12 to 15 days, which
II is incorrect. is inclined in a way that the telescope’s sky
109. Consider the following statements about view is largely free from obstructions by our
Dhruvastra Missile bright planet and the Moon. The prime
I. It is a long-range surface-to-surface mission ended on July 4, 2020, and TESS is now
helicopter-launched anti-tank guided on an extended mission. TESS is finding
missile (ATGM) system. planets ranging from small, rocky worlds to
giant planets, showcasing the diversity of
planets in the galaxy.
AMIGOS IAS 39
111. Consider the following statements about e- 2) Unmanned aerial vehicle (UAV) of Indian
Sanjivani Air force
I. It is a telemedicine app that provides both 3) Air Defence Fire Control Radar by DRDO
doctor-to-doctor and doctor-to-patient 4) A supercomputer by CSIR
telecommunication. Answer: 1
II. It is designed and developed by The Explanation:
Centre for Development and Advanced  ‘Neerakshi’ designed for mine detection
Computing (C-DAC). (currently in the prototype stage)
Which of the above given statements is/are Developed by: Garden Reach Shipbuilders
correct? and Engineers (GRSE) Ltd and AEPL, an
1) I only 2) II only MSME entity.
3) Both I and II 4) Neither I nor II  Features: 2.1-meter-long cylindrical AUV,
Answer: 3 weighing around 45 kg, can perform
Explanation: various tasks such as mine detection,
 It is a cloud-based integrated telemedicine disposal, and underwater surveys. It has
solution of the Ministry of Health and an endurance of four hours and a depth
Family Welfare, Govt. of India. It is a of up to 300 meters, making it a versatile
telemedicine app that provides both tool for ensuring coastal safety and
doctor-to-doctor and doctor to-patient protection.
telecommunication. Hence statement I is 113. Consider the following statements
correct. I. Aditya L1 is the first space-based Indian
 It is being designed, developed, deployed, mission to study the Sun.
and maintained by The Centre for II. The spacecraft shall be placed in a halo
Development and Advanced Computing orbit around the Lagrange point 1 (L1) of
(CDAC), Mohali. Hence statement II is the Sun-Earth system.
correct III. Lagrange points are specific locations in
Two modules of eSanjeevani: space where the gravitational forces of
 eSanjeevani AB-HWC: The doctor-to- two large bodies produce enhanced
doctor telemedicine platform, being regions of gravitational equilibrium.
implemented at all the Health and Which of the above given statements is/are
Wellness Centres (HWCs) in the country correct?
under the Ayushman Bharat (AB) Scheme 1) I and III 2) I and II
of Government of India. 3) II and III 4) I, II and III
 It operates on a Hub-and-Spoke model Answer: 4
wherein the ‘Ayushman Bharat-Health and Explanation:
Wellness Centres’ (HWCs) are set up at the  Aditya L1 is the first space-based Indian
state level, which is connected with the mission to study the Sun. It will be launched
hub (comprising MBBS/ Specialty/Super- by the PSLV-XL launch vehicle. The
Specialty doctors) at the zonal level. spacecraft shall be placed in a halo orbit
 eSanjeevani OPD: It is the Patient-to- around the Lagrange point 1 (L1) of the
Doctor remote consultation services rolled Sun-Earth system, which is about 1.5
out in 2020 amid the first lockdown million km from the Earth.
imposed to fight the Covid-19 pandemic,  A satellite placed in the halo orbit around
while the Outpatient Departments (OPDs) the L1 point has the major advantage of
in the country were closed. It enables continuously viewing the Sun without any
people to get outpatient services in the occultation/eclipses. This will provide a
confines of their homes. greater advantage in observing solar
112. Neerakshi, recently seen in news is related activities and their effect on space weather
to? in real-time.
1) Autonomous Underwater Vehicle (AUV)
AMIGOS IAS 40
 Lagrangian points, also known as Lagrange  NIDHI-Technology Business Incubator
points or libration points, are specific (TBI): Supports and nurtures knowledge-
locations in space where the gravitational driven innovative start-ups
forces of two large bodies, such as a planet  NIDHI-Seed Support Program (SSP):
and its moon or a planet and the Sun, Provides early-stage funding to promising
produce enhanced regions of gravitational start-ups
equilibrium. In these points, the  NIDHI-Center of Excellence (CoE): Creates a
gravitational pull from the two bodies world-class facility to help startups go
creates a stable or quasi-stable region global
where a third, smaller object can maintain
 NIDHI-Accelerator: Accelerates startups
a relatively constant position relative to the
through focused interventions.
larger bodies. There are five primary
115. Consider the following statements with
Lagrangian points, labeled L1 through L5, in
respect to ‘Gene-edited mustard’
a Sun-Earth system.
I. It is based on CRISPR/Cas9 gene editing
Hence all the three statements are correct.
while being non-genetically modified
114. NIDHI Program, recently seen in news is an
II. It is India’s first ever low pungent mustard
initiative of which of the following ministry?
that is pest and disease resistant
1) Ministry of Micro, Small and Medium
Which of the above given statements is/are
Enterprises
correct?
2) Ministry of Finance
1) I only 2) II only
3) Ministry of Science and Technology
3) Both I and II 4) Neither I nor II
4) Ministry of Commerce and Industry
Answer: 3
Answer: 3
Explanation:
Explanation:
 CRISPR/Cas9 – a gene-editing tool
 National Initiative for Developing and
deploying an enzyme was used, which acts
Harnessing Innovations (NIDHI) is an end-
as a “molecular scissors” to cut the DNA at
to-end plan for startups to double the
precise targeted locations of the gene, and
number of incubators and startups in the
then letting the natural DNA repair process
duration of five years.
to take over. The new GTR genes-edited
 Objectives: mustard lines are transgene-free or non-
 Promote S&T-based entrepreneurship and genetically modified (GM). Hence
startup ecosystem across the country statement I is correct.
Nurture start-ups through scouting,  Indian scientists have developed the first
supporting, and scaling innovations ever low pungent mustard that is pest and
Components disease-resistant. Hence Statement II is
 NIDHI-Entrepreneur in Residence (EIR): correct.
Inspires graduating students to pursue 116. Consider the following statements about
entrepreneurship through fellowships
 NIDHI-Promotion and Acceleration of Service Module Propulsion System (SMPS)
Young and Aspiring Technology I. The service module of Gaganyaan is a
Entrepreneurs (PRAYAS): Supports regulated tri-propellant based propulsion
innovators and startups from Idea to system.
Prototype stage II. The SMPS helps in performing orbital
 NIDHI-Inclusive-Technology Business injection of the spacecraft during the
Incubator (iTBI): Expands incubation ascent phase.
programs to various geographies and Which of the above given statements is/are
genders incorrect?
1) I only 2) II only
3) Both I and II 4) Neither I nor II

AMIGOS IAS 41
Answer: 1 Rule (8th Amendment), 1988 are the Acts
Explanation: and Rules that regulate GM Crops in India.
 The service module of Gaganyaan is a  Bt cotton is the only genetically modified
regulated bipropellant-based propulsion (GM) crop that has been approved for
system. Hence statement I is incorrect. commercial cultivation in 2002 by the
 It caters to the requirements of the Orbital Government of India. Long term studies
Module, performing orbit injection, were conducted by ICAR on the impact of Bt
circularisation, on-orbit control, de-boost cotton which did not show any adverse
manoeuvring, and SM-based abort (if any) effect on soil, microflora and animal health.
during the ascent phase.  Hence all the three statements are correct.
 The 440 N thrust LAM engines provide the 118. ‘SAGAR SAMPARK’ recently seen in news is a
main propulsive force during the ascending 1) Maritime expedition
phase, while the RCS thrusters ensure 2) Differential navigation satellite system
precise altitude correction. 3) Naval exercise
 With the successful completion of the hot 4) Security doctrine
test, the SMPS demonstrated its Answer: 2
performance in the full configuration. Explanation:
Hence statement II is correct.
SAGAR SAMPARK is an indigenous Differential
117. Consider the following statements with Global Navigation Satellite System (DGNSS).
respect to ‘GM (Genetically Modified) Crops’ The Differential Global Navigation Satellite
1. It is made using a technology that involves System is a terrestrial based enhancement
inserting DNA into the genome of an system which corrects the errors and
organism inaccuracies in the Global Navigation Satellite
2. They are regulated under the provisions of System (GNSS) allowing for more accurate
the Biodiversity Act, 2002 positioning information. Sagar Sampark at six
3. Bt cotton is the only genetically modified locations will assist the Director General of
(GM) crop that has been approved for Lighthouses and Lightships (DGLL) to provide
commercial cultivation by the radio aids to ships for marine navigation. The
Government of India system will provide accurate information to
How many of the statement(s) given above the ships for safe navigation and will reduce
are correct? the risk of collisions, groundings, and
1) Only one 2) Only two accidents in the port and harbour areas.
3) All three 4) None 119. Consider the following statements with
Answer:3 respect to ‘Chandrayaan-3 Mission’
Explanation: I. It consists of three payloads comprising of
a lander, rover and an orbiter module
 Genetic Modification is a technology that
involves inserting DNA into the genome of an II. It aims to conduct in-situ scientific
organism. To produce a GM plant, new DNA experiments on the lunar surface
is transferred into cells of a plant. These cells How many of the statement(s) given above
are then grown in tissue culture where they are correct?
transform into plants. The seeds produced 1) I and II 2) II only
by these plants will have new DNA. The most 3) I only 4) None
common way of inserting is using gene guns Answer: 2
method. Explanation:
 Environment Protection Act, 1986 (EPA),  Chandrayaan-3 is a planned third lunar
Biological Diversity Act, 2002, Plant exploration mission by the Indian Space
Quarantine Order, 2003, GM policy under Research Organisation (ISRO). It will consist
Foreign Trade Policy, Food Safety and of a lander and a rover similar to
Standards Act, 2006, Drugs and Cosmetics Chandrayaan-2, but would not have an

AMIGOS IAS 42
orbiter. Its propulsion module will behave
like a communication relay satellite. The
propulsion module will carry the lander and
rover configuration until the spacecraft is in
a 100 km lunar orbit. Hence statement I is
incorrect.
 Chandrayaan-3 will have three major
objectives. One is to demonstrate safe and
soft landing on the surface of the moon.
The second is to demonstrate rover
operations on the moon. And the third
objective is to conduct in-situ scientific
experiments on the lunar surface. Hence
statement II is correct.
120. Consider the following statements with
respect to ‘CE-20 Cryogenic Engine’
I. It will power the Cryogenic Upper Stage of
the LVM3 launch vehicle for the
Chandrayaan-3 mission.
II. It is designed by the Liquid Propulsion
Systems Centre (LPSC)
Which of the above given statements is/are
correct?
1) I only 2) II only
3) Both I and II 4) Neither I nor II
Answer: 3
Explanation:
 CE-20 cryogenic engine will power the
Cryogenic Upper Stage of the LVM3 launch
vehicle for the Chandrayaan-3 mission. The
high thrust cryogenic engine is the most
powerful upper stage cryogenic engine in
operational service. Hence statement I is
correct.
 It has been designed and developed by the
Liquid Propulsion Systems Centre (LPSC) a
subsidiary of ISRO. It is the first Indian
cryogenic engine to feature a gas-
generator cycle. Hence statement II is
correct.

AMIGOS IAS 43

You might also like